You are on page 1of 55

ICM Red Eye Qstream:

1.

A 73 y/o male presents with 3 months history of red eye with frequent tearing. He says the eye feels as
if it has something in it. The eyelid is shown below. Which of the following is most likely the diagnosis?

a.
b.
c.
d.
2.

Entropion
Ecropion
Pterygium
Stype

A 47 yr old man presents with the following lesion. The possible sequelae for this is:

a.
b.
c.
d.

This will not affect vision


Corneal ulcers
this will affect vision
this is assosiated with floaters
this is a pterygium which can extend over the corea and affect vision

3.

A 28-year-old woman presents with a three day history of a red eye. Which one of the following
features from the history indicates a diagnosis of uveitis?
a. Intense photophobia
b. Grittiness and discomfort, particularly around the lid margins and eyelash follicles
c. Purulent discharge from both eyes with the eyelids being stuck together in the mornings
d. Marked itchiness in both eyes
Uveitis (iritis) is an autoimmune reaction often precipitated by trauma. It causes red eye and photophobia

4.

A 62 yo man presents with an itchy, painful rash over his right forehead and eye lid shown here. What
feature would predict corneal involvement the most?

a.
b.
c.
d.
5.

extension to the opposite side of forehead


severity of rash
Involvement of eyelid
involvement of the nasocilliary branch of CNV

A 5 yr old child is brought in with the eye lesion shown below. Which of the following bacteria most
commonly causes this in children?

a.
b.
c.
d.

Staph aureas
HIB
Strep viridans
Clostridium difficile
This child has orbital cellulitis which is commonly caused by HIB in children
1

6.

A 66 yr old woman comes to see you with rapid onset red eye of 12 hours. She also notes a strange light
halo around objects in her vision and a headache. What is the likely diagnosis?
a. Open angle glaucoma
b. Phenytoin over dose
c. Closed angle glaucoma
d. Episcleritis
red eye, photophobia and a halo around lights are classic symptoms of acute closed angle glaucoma

7.

Which of the conditions listed below are a risk factor for acute closed angle glaucoma?
a. Reading in dim light
b. Hypertension
c. African heritage
d. Obesity
most of these are risk factors for open angle glaucoma except reading in dim light. Other RFs for acute
closed angle glaucoma are mydratic eye drops and other drugs which cause a dilated pupil.

8.

You are attending an ophthalmology clinic and are asked to look at the following retina. What is the
diagnosis?

a.
b.
c.
d.

Retinitis pigmentosa
Uveitis
Proliferative diabetic retinopathy
acute closed angle glaucoma

this is a classic picture of laser treatment for diabetic proliferation of blood vessels. These new vessels
interfere with vision and need to be removed by the laser.
9.

A 67 yr. old comes in with sudden onset blindness in his left eye. He denies any pain or tenderness over
his eye or face and states that the vision just clouded and disappeared over a few minutes this morning
when he was eating his breakfast. He has a past medical history significant for hypertension and he had
a small TIA last year. He takes lisinopril and aspirin daily. On exam he has a normal physical exam
including a normal neurological exam. Examination of his fundus reveals the following finding. What is the
likely cause of his loss of vision?

a.
b.
c.
d.

raised intracranial pressure


retinal vein occlusion
retinal artery occlusion
acute closed angle glaucoma

This patient has a sudden painless loss of vision. The causes for this can include retinal vein or artery
occlusion. Very occasionally severe MS might present like this as well. However with a patient with risk
factors for atherosclerosis (HT and Past TIA) and a retina with the classic dark and stormy
appearance, the diagnosis of retinal vein thrombosis is the best answer.

10. A 76 yr. old woman presents with sudden onset loss of vision in her right eye. Examination of her retina
shows the following: What symptoms may she have suffered from in the days before her loss of vision?

a.
b.
c.
d.

pain in the temple area


stroke(post circulatory)
seeing halo around lights
hypertension

This patient has a classic retina for a retinal artery occlusion. There is a paucity of vessels but a bright
red looking macule the classic cherry red macule. Hyper tension is a RF for retinal vein occlusion, a
posterior circulatory stroke would give a homonymous hemianopia vision field defect with macular
sparing, a halo suggests acute closed angle glaucoma and tunnel vision suggests chronic open angle
glaucoma none of which would cause monocular blindness so rapidly. However giant cell arteritis is
renowned for causing emboli which lodge in the retinal artery causing occlusion. It classically affects older
women and gives tenderness to palpation over the temple. She might also have noticed jaw claudication.

ICMII Neuro Qstream:


1.

A 24 year old secretary comes to see you complaining of becoming increasingly clumsy and weak over the
past 6 months. She finds that she cannot type as accurately or fast as before and often drops things for
no reason. On examination you note a well looking young woman with Bp 120/80, pulse 84 regular, BMI
25. She has reduced tone and reflexes in both upper limbs while her knee reflexes are very brisk with 7
beats of clonus in the left ankle although normal clonus in the right ankle. She has also developed some
difficulty in speaking and swallowing. She has remained afebrile and denies any myalgias or athralgias.
She has normal sensation with no dissociated sensory loss. What is the likely diagnosis?
a. Multiple sclerosis
b. Guillain barre syndrome
c. Amyotrophic lateral sclerosis
d. Werdnig-Hoffman disease
This is a mixture of upper and lower motor neuron signs typical of ALS

2.

A 50 yr old man presents with a dry cough, hoarseness, wheezing and pain in the right chest for the
past 4 weeks. He had previously had a mildly productive cough on and off for the past 5 years. He is a
smoker, and has been smoking a pack of cigarettes per day since age 12. Physical examination reveals
a man with a wasted general appearance, and with both crackles and wheezing on auscultation of the
right lung. Physical examination is also significant for weakness during flexion, extension, adduction, and
abduction of the upper limbs at the shoulder. Chest x-ray shows a central mass in the middle lobe of the
right lung. Which of the following mechanisms best explains this patients muscle weakness?
a. Production of auto-antibodies against pre-synaptic Ca channels
b. Compression of the lower trunk of the brachial plexus
c. Compression of the superior vena cava
d. Excessive secretion of ADH leading to hyponatremia
Clinical manifestations, age, past medical history of smoking suggest lung cancer. CXR findings of a
central lung mass and weakness of the shoulder (proximal muscles of the upper limb) suggest small cell
lung cancer and Lambert-Eaton syndrome respectively. Lambert-Eaton syndrome is characterized by
production of auto-antibodies against pre-synaptic Ca channels at neuromuscular junctions. This leads to
neuromuscular block and subsequent weakness of the affected muscles

3.

A 53 year old man develops tremor in his fingers. The tremor is most pronounced when he sits with his
hands resting in his lap. Which of the following components of the nervous system is most likely to be
involved?
a. VPL
b. Cerebellar hemisphere
c. Cerebellar vermis
d. Substantia nigra

4.

A 43-year-old woman develops a progressive, ascending motor weakness over the past 4 days. She is
afebrile. She is hospitalized and requires intubation with mechanical ventilation. A lumbar puncture is
performed with normal opening pressure. The CSF obtained is clear and colourless with glucose of 65
mg/dL (serum 100 mg/dL), protein 95 mg/dL, and cell count of 5/microliter, all lymphocytes. She
gradually recovers over the next month. Which of the following conditions most likely preceded the onset
of her neurological disease?
a. Vitamin B12 deficiency
b. Systemic lupus erythematosus
c. Viral pneumonia
d. Ketoacidosis
This is a classic picture of ascending motor weakness and reduced reflexes typical of Guillain Barre. GB is
most commonly preceded by a viral infection.

5.

An 80 yr old man has small irregular pupils that constrict to near vision but not to light. He has been in a
state hospital for the past 20 yrs because of dementia and marked slowing of his thought processes.
Where would you best localize the lesion accounting for his pupillary abnormality?
a. Pretectum
b. Ciliary ganglion
c. Nucleus of Edinger-Westphal
d. Optic nerve

6.

A 50 yr old male with long standing hypertension & Type 1 diabetes, is brought to the ER following a
motor traffic accident. His car skidded on ice and hit the concrete base of a metal lamppost. The
paramedics had to call in the jaws of life to help extricate the patient from the car. On arrival he is
conscious and restrained on a back board with an immobilized neck. On examination he has power of 0/5
on his left arm and left leg with absent reflexes in his arm. His left leg shows equivocal reflexes with up
going planter on the left. He shows a dissociated sensory loss with absent vibration and proprioception on
the left and absent pain and temperature on the right. He is sent for an immediate MRI of the brain and
spinal cord. What are the likely findings which could explain these signs?
a. Destruction of the posterior columns of the spinal cord
b. An enlarged central syringx in the cord
c. Hemisection of the spinal cord
d. Infarction of the medulla
Loss of ipsilateral corticospinal tracts & dorsal columns, with contralateral spinothalamics is classic of
Brown Sequard or hemi section of the cord. Trauma is the commonest etiology of this type of injury.

7.

A 21 yr old woman presents to your office with visual difficulty for the past 5 days. She notes that at
times colors appear washed out, especially following a hot shower, and that looking around has sometimes
been uncomfortable. She describes intermittent back pain which she cant understand as she doesnt
recall any trauma or an inciting incident. Occasionally the pain shoots down her entire back like an
electric shock. Her right hand tingles intermittently in the morning. She was partially symptomatic last
august when she had episodes of severe fatigue and she fell several times during soccer practice which
attributed this to her long layoff during the preceding summer months that left her out of shape. On
examination visual acuity is 20/60 in her right eye and 20/20 in her left. There is mild pallor of the
right optic disc. On testing the cardinal directions of gaze you notice that her right eye doesnt move all
the way medially with left lateral gaze. Finger nose testing reveals subtle bilateral abnormalities as
does heel-to-shin testing on the right. Flexion of her neck can reproduce the painful sensation described
above. Damage to which of the following areas best explains the eye movement abnormality?
a. Oculomotor nucleus
b. Superior colliculus
c. Medial longitudinal fasciculus
d. Medial lemniscus

8.

A 73-year-old man has exhibited problems remembering things for the past 7 months, and he is noted by
his immediate family to confabulate. He dies as a consequence of a hepatocellular carcinoma. At autopsy,
his brain demonstrates bilaterally small mamillary bodies that show brown discoloration. Microscopically,
there is gliosis and vascular proliferation and hemosiderin deposition in the mamillary bodies and
periaqueductal gray matter. Which of the following is the most likely diagnosis?
a. Huntington Disease
b. Wernicke-Korsakoff syndrome
c. Parkinsons Disease
d. Multiple sclerosis

9.

Numerous inherited disorders are the results of the expansion of trinucleotide repeats either within the
coding regions of the genes or the untranslated regions of the resultant RNAs. Which of the following
conditions have been shown to be caused by triplet expansion?
a. Erlos Danlos syndrome
b. Huntingtons disease
c. Rotors syndrome
d. Duchenne muscular dystrophy

Trinucleotide repeats are seen in HD. The length of the repeating region gets longer from generation to
generation and is associated with younger onset of symptoms.
10. A 77-year-old man has had a step-wise loss of mental function for the past 8 years, but his condition
has remained stable for the past year. He can no longer care for himself. On physical examination he has
decreased motor strength in his right upper extremity. He is able to ambulate and his gait is normal. He
cannot remember any of 3 objects after 3 minutes. A chest x-ray shows cardiomegaly. Laboratory
studies show serum glucose of 95 mg/dL, cholesterol 192 mg/dL, creatinine 1.0 mg/dL, total thyroxine
9.5 microgm/dL, vitamin B12 481 pg/mL, and hemoglobin 13.9 g/dL. Which of the following is the
most likely diagnosis?
a. Diffuse Lewy body dementia
b. Alzheimers disease
c. Vascular dementia
d. Huntington disease
This is the classic step wise deterioration in mental functioning associated with vascular dementia.
11. A 24 year old primary school teacher presents with diplopia. On examination you note that looking to the
left, her left eye abducts normally but her right eye remains in the midline. convergence is unaffected.
This condition is often caused by damage to which of the following cells?
a. Schwann cells
b. Oligodendrocytes
c. Purkinje cells
d. Microglial cells
This patient has internuclear opthalmoplegia caused by a lesion in the MLF. This is a common symptom of
multiple sclerosis caused by damage to oligodendrocytes.
12. A 44 year old woman with a past medical history of rheumatic fever and classical migraine presents in a
coma with a GCS of 10. She shows generalized hyper-reflexia and decerebrate posturing to a painful
stimulus. 10 days later it is noted that she can move her extraocular muscles and seems to be able to
reliably answer questions by moving her eyes. However she is unable to initiate any other voluntary
movements. Which of the following vessels is likely to be occluded in this patient?
a. Anterior cerebral artery
b. Vertebrobasilar artery
c. Posterior inferior cerebellar artery
d. Middle cerebral artery
This is a typical locked-in patient caused by a large infarction of the brain stem. The vessel large enough
to cause the extensive infarction is the vertebrobasilar artery.
13. A 10-month-old infant is noted to be failing to reach developmental milestones. On physical examination
there is a prominent 2 cm lumbar meningomyelocele. An MRI scan of the brain demonstrates downward
extension of the cerebellar vermis and displacement of the medulla from a small posterior fossa into the
foramen magnum. There is tenting of the tectum of the midbrain. The cerebral ventricles are enlarged.
The spinal cord has findings of hydromyelia. Which of the following is the most likely diagnosis?
a. Maternal folate deficiency
b. Arnold Chiari malformation
c. Viral encephalitis
d. Dandy-Walker malformation
Arold chiari malformation is characterized by extension of the cerebellar vermis through the foramen
magnum which can block CSF flow causing hydrocephalus. It can cause syringomyelia as well as being
associated with spina bifida.

14. You have examined a 72 year old man who was suspected of having a cerebrovascular accident several
weeks earlier. You find that his upper extremities show wasting of muscles, paralysis, and areflexia, but
his lower extremities, although paralyzed, are not wasted, and show hyperreflexia and clonus. He has loss
of pain and temperature sensation over both arms and legs but proprioception and vibration sensation is
normal. The problem is consistent with an infarct in:
a. Middle cerebral artery
b. These findings are not consistent with any possible pattern of a vascular infract
c. Anterior spinal artery to cervical cord
d. Basilar artery to the brain stem
Anterior cord syndrome characterized by loss of motor function below the level of injury, loss of
sensations carried by the anterior columns of the spinal cord (pain and temperature), and preservation of
sensations carried by the posterior columns (fine touch and proprioception).
15. A 65 year old left handed man with a history of hypertension and diabetes mellitus presented to the
Emergency Department complaining of dizziness and lack of coordination in his right arm. He awoke that
morning as if the room were spinning. He managed to shower although he nearly fell in the shower stall
because of dizziness. Shaving with his left hand proved difficult. Although he was nauseated, he tried to
eat a bowl of cereal. When he managed to get food into his mouth he repeatedly choked on it. He then
called 911. His neurological exam findings were as follows: He had slight right ptosis and the right pupil
was smaller than the left. Eye movements were full but significant horizontal nystagmus was present.
Perception of pin prick was decreased over the entire right face. The palate did not elevate at all on the
right and the gag reflex was diminished. He was hoarse and had dysarthric speech. No muscle weakness
was noted. He had ataxia with finger-nose-nose testing on the right.
a. Right cerebral hemisphere
b. Right lateral medulla
c. Left lateral medulla
d. Right medial pons
This is a PICA stroke affecting the lateral medulla. CNs do not decussate thus right eye affected indicates
a right sided lesion
16. An 18-year-old army reservist is brought to the local barracks physician with lethargy, a high-grade
fever and a headache. O/E nuchal rigidity is present. A lumbar puncture (LP) is performed and
demonstrates increased protein and decreased glucose in the CSF. The neutrophil counts are increased and
lymphocyte counts are within normal limits. What is the most likely organism responsible for this
patients condition?
a. Escherichia coli
b. Streptococcus pneumoniae
c. Neisseria meningitides
d. Cytomegalovirus
Meningitis in the stated age group with increased neutrophils in the CSF is likely caused by Neisseria
meningitides
17. A 55 year old alcoholic male presents to the emergency room with ataxia, global confusion and psychosis.
He also has polyneuropathy and opthalmoplegia. Which of the following would be the most suitable
therapy for this patient?
a. Niacin
b. Biotin
c. Thiamine
d. Riboflavin
This patient has Wernickes encephalopathy presenting with confusion, ataxia and opthalmoplegia. This is
caused by thiamine deficiency.

18. An 85 year old man presents with abrupt onset of talking gibberish according to his wife. On
examination, his speech is fluent with normal prosody, but there is no meaningful content. What
associated neurologic deficit would you expect?
a. Left hemiparesis
b. Right homonymous hemianopia
c. Right hemiparesis, leg greater than arm
d. Anosognosia
The optic radiation travels in the same region as Wernickes area of speech
19. A 43 yr old man comes to see you complaining of sensation changes and weakness in his arms. He is
asthmatic and uses salbutamol and inhales steroids daily. He also gives the history of a major road traffic
accident 10 yrs ago which left him with whiplash injuries and required 6 months off work. However he
made a full recovery. On exam you note loss of temperature and pain sensation over his deltoid muscles
and arms bilaterally. He also has weakness in his left arm with reduced reflexes in his left biceps and
triceps muscles. You suspect syringomyelia and refer him to the local neurologist for further evaluation.
On exam of the patient by the specialist what other signs might be noted?
a. Increased reflexes in his right arm
b. Loss of pain and temperature sensation to his lower limb girdle
c. Loss of joint position sense in all four limbs
d. Unilateral ptosis and miosis
syringomyelia in the cervical cord can cause a horners syndrome by extension of the syrinx as far as the
lateral cord where is damages the pre-ganglionic sympathetic neurons. This may be a symmetrical or
asymmetrical loss.
20. A 25 year old woman comes to you asking for genetic testing because of an inherited neurological
condition. She describes members of her family _ both male and female developing writhing arm
movements and a dancing gait, together with a serious dementia. These symptoms start in late middle
age and result in the early death of her relatives. You send her for genetic testing what does the
geneticist look for?
a. Karyotype 45XXY
b. 22q11.2 deletion
c. Multiple CAG repeats
d. Karyotype 47xx
trinucleotide repeats are found in Huntingtons as well as other conditions such as Friedrichs ataxia. The
patient has HD.
21. A 44 year old man was hit on the lateral right side of the skull while playing Golf. He lost consciousness,
but quickly improved without any intervention. He continues to play for half an hour but then began to
complain of a severe headache and he started vomiting. He is rushed to the Emergency room where he
collapses unconscious during assessment. On examination you note the patient is in a deep coma. Pupil
exam reveals that his right sided pupil is dilated and sluggishly reactive to light. CT scan of his brain is
ahown below. What is the most likely diagnosis?

a.
b.
c.
d.

Subdural Hematoma
Subarachnoid Hemorrhage
Intraparenchymal Hemorrhage
Epidural Hematoma

Epidural hematoma is from bleeding of the middle meningeal artery. Patients typicaly have a lucid interval
8

followed by deterioration and loss of consiousness again. The CT scan shows a convex shaped haematoma
as illustrated above.
22. A 55 yr old man is brought in by his wife as she is concerned about his personality changes exhibited over
the past year. He used to be a hard working and conscientious man but recently he has become slovenly
and aggressive. His wife discovered that he had been fired from his job after 18 yrs because for swearing
at a colleague and sexually propositioning a secretary. On exam you note the following vitals: Bp of
130/85, pulse 77 regular, temp 36.5C. No abnormalities found on examination but he scores only 18 on
mini-mental state exam. You arrange an MRI scan that shows shrinking of the frontotemporal lobes.
What is his likely diagnosis?
a. Picks disease
b. Vascular dementia
c. Alzheimers disease
d. Huntingdons disease
Picks disease or fronto-temporal dementia is a genetic disease characterized by early onset personality
changes.
23. A 60 yr old hypertensive male, presents with severe headache and drowsiness. On further inquiry his
wife states he has not been himself for several weeks. In particular he lacks concentration and has been
irritable. You note that he smells of alcohol. When you ask his wife she admits that he drinks almost a
bottle of whiskey per day. On examination he is stuporous with dysarthria and a blood pressure of
160/120 mm of Hg. A CT scan of the brain is shown below. What is the most likely source of bleeding?

a.
b.
c.
d.

Middle Meningeal Artery


Posterior Communicating Artery
Bridging Veins
Anterior Communicating Artery

This is a subdural hematoma usually from a trivial head injury that is often forgotten. The bleeding is from
the bridging veins and shows up on the CT scan as a crescentic opacity.
24. A 45 year old man with polycystic kidney disease is rushed to the emergency room complaining of a
sudden onset severe headache. He says he first thought someone had hit him on the back of the head.
Upon examination there is slight nuchal rigidity with B.P 160/98mm Hg, pulse rate of 96/min, and
temperature 98.4 F. An emergency non-contrast CT scan of the brain was done. Lumbar Puncture
reveals xanthochromia. What is the most likely diagnosis?
a. Epidural Hematoma
b. Meningioma
c. Subarachnoid Hemorrhage
d. Multi-infarct Dementia
The onset of severe (worst headache of his life) in a patient with PKD suggests a ruptured Berry aneurysm
resulting in a subarachnoid bleed. CSF is typicaly xanthochromic

25. A 21 yr old woman presents to your office with visual difficulty for the past 5 days. She notes that at
times colors appear washed out, especially following a hot shower, and that looking around has sometimes
been uncomfortable. She describes intermittent back pain which she cant understand as she doesnt
recall any trauma or an inciting incident. Occasionally the pain shoots down her entire back like an
electric shock. Her right hand tingles intermittently in the morning. She was partially symptomatic last
august when she had episodes of severe fatigue and she fell several times during soccer practice which
attributed this to her long layoff during the preceding summer months that left her out of shape. On
examination visual acuity is 20/60 in her right eye and 20/20 in her left. There is mild pallor of the
right optic disc. On testing the cardinal directions of gaze you notice that her right eye doesnt move all
the way medially with left lateral gaze. Finger nose testing reveals subtle bilateral abnormalities as
does heel-to-shin testing on the right. Flexion of her neck can reproduce the painful sensation described
above. An MRI of the brain and spinal cord is most likely to show which of the following?
a. Enhancing mass lesion in the belly of the pons
b. Spinal cord and brain stem white matter lesions
c. normal findings
d. Scattered subcortical white matter lesions
26. A 72-year-old woman has a five year history of worsening mental functioning with trouble
remembering things. Her husband notices that she has developed a hand tremor and is falling often. MR
imaging of the brain shows symmetrically increased size of the lateral ventricles along with cerebral
cortical atrophy in a mainly frontal and parietal distribution. A lumbar puncture reveals a normal opening
pressure, and analysis of the clear, colorless cerebrospinal fluid reveals a glucose and protein which are in
normal ranges. Cell count on the CSF shows 3 WBCs (all lymphocytes) and 1 RBC. A funduscopic
examination is normal. Which of the following abnormalities is most likely to be associated with her
underlying disease process?
a. Loss of gamma aminobutyric acid
b. Loss of Betz cells
c. increased numbers of Lewy bodies
d. Perivascular mononuclear inflammation
27. A 2 year old Nigerian child is brought to the Emergency room by his mother. Through an interpreter you
discover that this is the third time the child has been brought in over the past 9 days but each time the
child is discharged with no diagnosis. The mother gives a history of the child having a mild cold with one
day of fever and a runny nose 11 days ago. Since then the child has been increasingly sleepy and off his
food. The child can be roused from sleep but is very cranky and irritable. The child has vomited once 2
days ago after a lot of crying but has been taking fluids and milk since then without difficulty. On
examination you note temp 38C, Pulse 130 bpm, resp 22. All examination systems are normal. Peripheral
blood film reveals an increased neutrophil count. Cerebrospinal fluid has reduced glucose, increased protein
and increased neutrophills. What is the likely diagnosis?
a. Strep pneumonia meningitis
b. Lead poisoning
c. Neisseria meningitides
d. Haemophilus influenza Type B meningitis
28. A 79-year-old man comes to establish primary care. He is accompanied by his daughter. He has been in
very good health for most of his life. His only medication is a beta blocker, which he has been taking for
moderate hypertension. He has undergone all screening examinations appropriate for his age. Over the
past several months, he underwent evaluation for possible onset of dementia. The patient and his
daughter agree that the patient's memory has been worsening over the past 1 to 2 years. He easily
recalls events of his childhood, but he is not able to tell you what he ate for his morning meal. He had been
an avid outdoorsman, but he had to give up outdoor activities because he recently got lost in the woods for
several hours. The family seeks your opinion and further workup. Which of the following statements
regarding Alzheimer disease (AD) is true?
a. The definition of AD requires impairment in only one area of cognitive function
b. The pivotal cognitive finding in AD is retrograde amnesia
c. AD is rapidly fatal and patients have a life expectancy of only 18 months.
d. AD is histopathologically defined by neurofibrillary tangles and neuritic plaques in the cerebral
cortex

10

29. A 55 years old male complaints of vertigo, defective vision, weakness of his right side and difficulty in
swallowing. Physical examination reveals right hemiparesis, dysarthria and right homonymous
hemianopia with macular sparing. This suggests a defect in primary visual area due to occlusion of which
of the following vessels?
a. Right Posterior cerebral artery
b. Left Post Cerebral artery
c. Left Middle cerebral artery
d. Left Anterior cerebral artery
Occipital lobe is the primary visual cortex supplied by Posterior cerebral artery. Right temporal field of
vision is carried by the right nasal retinal fibers and these will cross over to opposite side at optic chiasm
and continue as optic radiation to occipital lobe. Macular sparing is due to dual blood supply. Occlusion of
the posterior cerebral artery causes visual field defect right-sided homonymous hemianopia with macular
sparing
30. A 19 yr old student is receiving chemotherapy for his Hodgekin s lymphoma. He is also receiving
medication for a nausea. He is brought into the E.R suffering from an acute reaction. This involves his
eyes looking up and to the left and his back going into spasms. Which medication is she likely to be taking
for his nausea?
a. Gaviscon
b. Cimetadine
c. Omeprazole
d. Metoclopromide
31. A 6 yr old boy is referred from school with inattention and poor school performance. The child had a
normal antenatal care and normal vaginal delivery with no post natal complications. You hear that in
class he often seems to stop what he is doing and looks vacant or chews automatically for a few seconds
before the episode stops. You diagnose absence seizures. What medication is most appropriate first line
for absence seizures?
a. Methylphenidate
b. Ethosuximide
c. Phenobarbitol
d. Lorazepam
Ethosuximide is first line treatment for absence seizures
32. On a ward round in the neurosurgery department you are shown the following patient. What is the mode
of inheritance of this condition?

a.
b.
c.
d.

Continuous gene deletion


Single gene autosomal recessive
Single gene autosomal dominant
Single gene X linked
the patient has neurofibromatosis which is inherited as AD

11

33. A 32 yr old male with trisomy 21 suffers from increasing confusion and memory loss over several years.
Eventually he is unable to self care. Cortical neurons are most likely to show
a. Lewy bodies
b. Neurofibrillary tangles
c. Pick bodies
d. Negri bodies
Alzheimers dementia is commonly associated with Trisomy 21. Alzheimers dementia is associated with
neurofibrilary tangles
34. A 4 year old child is noted to have a head circumference on the 97th percentile. CT sows enlarged lateral
ventricles. What is the most likely long term complication of this condition
a. Infection
b. muscle hypertonicity
c. tumor
d. Sub-arachnoid haemorrhage
this child has hydrocephalus which causes stretching of the pyramidal tract and corresponding UMn
symptoms
35. A 65 yr old man presents to the E.R department with difficulty speaking. When he is asked a question he
tries to answer but is very hesitant in his speech, cannot form the words properly or get the right word.
He gives up in frustration. This condition is due to a lesion in:
a. Superior frontal gyrus on non dominant hemisphere
b. Inferior frontal gyrus of non dominant hemisphere
c. Superior frontal gyrus on dominant hemisphere
d. Inferior frontal gyrus of dominant hemisphere
this patient has an expressive dysphasia due to damage in Brocas areas which is found at the inferior
frontal gyrus of the dominant hemisphere
36. A 45 yr old patient presents with rapid onset dementia. On examination you note she has restless,
undulating movements of both hands. The family says that her mother had the same condition at around
the same age. Which part of the patients brain will show atrophy?
a. Frontal cortex
b. Substantia nigra
c. Putamen
d. Caudate nucleus
This patient has Huntingdons chorea which is characterized by atrophy of the caudate nucleus and
choreaform dance like involuntary limb movements
37. A 34 yr old swimmer presents with an external ear infection. During examination of the outer auditory
meatus with an ophthalmoscope the patient becomes very pale and faints. Which nerve has mediated this
event?
a. Trigeminal
b. Facial
c. Glossopharygeal
d. vagus
Posterior part of external meatus is supplied by the vagal nerve. This patient has had a vasovagal faint due
to stimulation of the vagal nerve.

12

38. A 35 yr old man comes complaining of a strange, tingling feeling in his hand over his fifth digit.
Sensation here is mediated through which nerve?
a. axillary
b. median
c. ulnar
d. radial
39. A 55 yr old homeless man is brought into the ER intoxicated. He is rehydrated with IV fluids and
dextrose. Over the next two days the physician notices that he has difficulty with his short term
memory, has nystagmus and opthalmoplegia and walks with a widebased, unsteady gait. Where is the
site of his pathology?
a. midbrain
b. caudate nucleus
c. pons
d. mamillary bodies
thiamine deficicy causes wernickes encephalopathy and Korsakoffs psychosis. This is from atrophy of the
mamillary bodies.
40. A 65 yr old man notices a mass in his tongue which is 1cm anterior to the foramen cecum in the midline.
What is the source of gustatory sensation in this area?
a. Glossopharyngeal nerve
b. Chorda Tympani
c. Vagus nerve
d. Trigeminal nerve maxillary branch
Taste in the tongue anterior 2/3 is served by the chorda tympani branch of the facial nerve. The posterior
1/3 is served by the Glossopharyngeal nerve. Sensation to the anterior 2/3 of the tongue (touch) is
served by the mandibular branch of the trigeminal nerve. Touch in the posterior 1/3 is via the
glossopharyngeal nerve.
41. A 34 yr old epileptic man presents with the following abnormality of his gums. Which drug is he taking?

a.
b.
c.
d.

phenytoin
clozapine
lithium
prednisolone

42. A 23 yr old woman comes in complaining of extreme weakness in both legs. On taking a history she tells
you that she has just broken up with her fiance. Examination reveals normal sensation and normal
reflexes and tone globally. An MRI is normal. What is the likely cause of her condition?
a. Malingering
b. Panic disorder
c. conversion disorder
d. somatisation
e. hypochondriac
43. A 29 yr old man complains of pain and decreased vision in his right eye for the past two days. His vision
has deteriorated to seeing movement only. He also has an intention tremor and a wide based unsteady
gait. His underlying condition is caused by:
a. intoxication
13

b.
c.
d.
e.

autoimmune
virus
bacteria
Continuous gene deletion

This patient has cerebellar signs and visual symptoms of optic neuritis suggesting Multiple sclerosis.MS is
an autoimmune disease.
44. A 15 yr old bay is brought to see you with gait instability and frequent falls. On examination you note he
has a Kyphosis, pes cavus, ataxia and reduced joint position and proprioception sensation. This patient has
an inherited condition which causes death through:
a. cardiomyopathy
b. renal failure
c. liver failure
d. aortic disease
this patient has Freidrichs ataxia, an inherited form of cerebellar dysfunction which affects boys 5-15
yrs old. It also leads to cardiomyopathy and diabetes
45. A patient is being evaluated for hypoventilation. He is asked to take deep breaths in rapid succession. The
physiologists measure the nerve activity and the pleural pressure changes and note the following:
Phrenic nerve
+++++++
++++++
+++++++
Pleural pressure
-10 cm water
-8
-4
Which of the following is the site of lesion? His problem is in which part of the nervous system?
a. spinal cord
b. phrenic nerve
c. brain stem
d. Neuromuscular junction
this patient has reduced response to the nerve firing which is a classical finding of myasthenia gravis. This
is a problem at the NMJ
46. A 43 yr old male smoker presents with double vision on walking downstairs. Which nerve is likely to be
affected?
a. optic nerve
b. oculomotor nerve
c. Trochlear nerve
d. Abducens nerve
The Trochlear nerve (IV cranial nerve) is responsible for intorsion of the eyeballs
47. A 40 yr old comes in with involuntary grimacing and strange dancing movements. The neuro exam
shows normal sensation, tone and reflexes. His father and grandfather had the same condition but it
started considerably later in life. This patient is demonstrating the condition earlier due to:
a. Increased penetrance
b. Pleitropy
c. Anticipation
d. Mosaicism
This patient has Huntingtons which comes on earlier in successive generations due to the phenomenon of
gene anticipation.

14

48. You admit a 78 year old man from the emergency department whose mobility has gradually deteriorated
over the last few weeks. He has no recollection of falling. On examination, he is disorientated and has a
left hemiparesis (power 4 out of 5). You request a CT scan of his brain. which of the following is the most
likely diagnosis?

a.
b.
c.
d.

epidural haematoma
Subdural Hematoma
subarachnoid hemorhage
intracerebral haematoma

The patients history of gradual onset neurological symptoms with confusion suggests a diagnosis of
subdural haematoma. The diagnosis of bilateral subdural haematomas is confirmed by the appearance of a
peripheral rim of blood under the skull, visible on the CT scan.
49. You see a 33 year old woman in the emergency department who has developed worsening weakness of
her right side and has had difficulty finding words over the past few days. She had an episode of vertigo
about four years ago, but has no other past medical history and has never smoked. A recent blood test
showed that her total cholesterol level was 4.0 mmol/l. An MRI shows the following:Examination of her
CSF is likely to show:

a.
b.
c.
d.

Oligoclonal bands
Bence Jones proteins
Reduced glucose and increased protein
Polyclonal antibody bands

The time course of the patients symptoms suggests a diagnosis of multiple sclerosis. You should refer the
patient to a neurologist so that they can confirm or refute this diagnosis. The diagnosis of multiple sclerosis
is based on finding two or more characteristic neurological events separated in space and time. Specialist
diagnosis looks for classic findings on MRI together with oligoclonal bands in the CSF.
50. A 76 yr old man is brought in with hearing difficulties. He complains of noting a whistling sound in his
right ear which interferes with his hearing . Tympanometry reveals reduced hearing acuity on the
right. You suspect sensorineural hearing loss. Which of the following results would confirm this
a. Sound lateralizes to the left with BC>AC on the left
b. Sound lateralizes to the left with AC>BC on the left
c. Sound lateralizes to the right with AC>BC on the right
d. Sound lateralizes to the left with BC>AC on the right
This patient would have a sensorineural deafness. In this situation bone conduction < air conduction so
Webbers lateralizes to the normal side and Rinnes reveals BC < air conduction

15

Skin & Connective Tissue Diseases Qstream:


1.

A 30 yrs old lady complains of itchy lesions on the extensor aspect of her knees, elbows and scalp. On
examination you find dry silvery scales and salmon colored plaques as shown in the picture. Her mother
had the same condition in the past. Which of the following is the most likely diagnosis?

a.
b.
c.
d.
e.

Dermatitis herpetiformis
Atopic dermatitis
Pemphigus
Actinic keratosis
Psoriasis

Psoriasis is an autoimmune disease of keratin synthesis affecting keratinocytes characterised by


hyperplasia.The most common areas of eruption include the elbows, knees, gluteal cleft and scalp. A
frequently seen complication is deforming joint disease. Pitting/crumbling of nails is often seen.
Histologicaly it is characterised by hyperparakeratosis, acanthosis(increase in thickness of stratum
spinosum) and Munro microabscesses.
2.

An 8 yrs old girl presents with failure to grow and a pruritic skin rash over her elbows and knees for 6
months. she was diagnosed with malabsorption syndrome 2 yrs ago. Examination reveals a
papulovesicular skin rash with blistering on the extensor aspects of her elbows and knees. Which of the
following is the most likely diagnosis?
a. Pemphigus
b. Dermatitis herpetiformis
c. atopic dermatitis
d. Psoriasis
e. Herpes zoster
Dermatitis herpetiformis is strongly assosiated with celiac disease (glutein sensitive enteropathy) and both
the skin condition and enteropathy improve with a glutein free diet. The lesions resemble those of herpes as
they are pruritic vesicles hence the name herpetiformis.

3.

A 2 yrs old girl is brought to the pediatrician with a rash on the face, diaper area and flexural aspects of
her limbs. She also has a history of recurrent wheezing since she was one yr old. Examination reveals
weeping papules and plaques on the stated areas as shown below. Which of the following is the most
likely diagnosis?

a.
b.
c.
d.

Atopic dermatitis
viral infection
Impetigo
Acne vulgaris

atopic dermatitis is a common inflamatory disease of child hood. Presents with pruritis and erythematous
weeping papules that occur in response to certain environmental antigens. patients have high serum levels
of IgE. The atopic triad constitutes rash,asthma and hayfever.

16

4.

A 50 yrs old male presents with Velvety thickening and darkening of his skin around the neck, axillae
and body folds as shown in the picture. Which of the following conditions may also be assosiated with
this disease?

a.
b.
c.
d.

Gastric cancer
Inflamatory bowel disease
Celiac disease
Arthropathy

Acanthosis nigricans presents as hyperpigmented plaques with a velvety texture in the skin fold areas.
The benign form is assosiated with Insulin resistance and the malignant form with GIT malignancy
5.

A 50 yrs old farmer presents to his primary care physician with history of a pearly nodule on his face
for the last 6 months. This nodule apears translucent with a central depressed area that easily bleeds to
touch. Telangiactasia(dilated capillaries) is seen surrounding the lesion. Microscopy of a punch biopsy
reveals nests of cells with a pallisading growth pattern. Based on this description and the image below
what is the most likely diagnosis?

a.
b.
c.
d.

Squamous cell carcinoma


Actinic keratosis
Malignant melanoma
Basal cell carcinoma

Basal cell carcinoma, Squamous cell carcinoma and malignant melanoma are the skin cancers that occur in
fair skinned individuals exposed to excessive UV radiation. The typical description of this skin nodule and
pallisading nests of cells suggest a Basal cell carcinoma diagnosis. Squamous cell carcinoma would present
as a cauliflower like growth on sun exposed areas and biopsy reveals keratin pearls
6.

A 45 yrs old man presents to his primary care physician with history of skin blisters. Examination
reveals bullous, fluid filled lesions as shown in the picture. Mucus membranes are seen to be normal.This
condition is characterised by the presence of IgG antibodies directed against which of the following?

a.
b.
c.
d.

Desmosomes
melanocytes
keratinocytes
Hemidesmosomes

Bullous pemphigoid is an autoimmune disease characterised by the presence of antibodies that attack
hemidesmosomes along the basement membrane of the dermoepidermal junctions. clinicaly the lesions are
serous fluid filled tense bullae mainly in the flexural aspects.mucous membranes are not involved

17

7.

A 70 yrs old man presents to his primary care physician with severe itching, burning and a sharp pain of
his left chest wall. He also has a rash in this area. Physical examination reveals several groups of vesicles
that extend from the left midaxillary line to the midline of the patients back as shown below. What is
the most likely diagnosis?

a.
b.
c.
d.

Chickenpox
Atopic dermatitis
Herpes zoster
Dermatitis herpetiformis

A vesicular rash with a unilateral dermatomal distribution is typical of shingles. Reactivation of the
varicella zoster virus that is lying dormant in the dorsal root ganglion occurs in the elderly or
immunocomromised patient as a result of an ineffective immune system. Biopsy would reveal
multinucleated giant cells (positive Tzank smear). Varicella zoster is the same virus that causes chickenpox.
8.

A 60 yrs old farmer develops a papular, erythematous, rough, sand paper like rash on his arms as shown
in the picture below. Biopsy reveals basal cell layer atypia with hyperkeratosis and para keratosis. This
patient is at a greater risk of developing which of the following conditions?

a.
b.
c.
d.

Basal cell carcinoma


sqaumous cell carcinoma
Seborheac keratosis
Actinic keratosis

The lesions described in this vignette suggest a diagnosis of Actinic keratosis due to exposure to sunlight
and has a potential for developing into squamous cell carcinoma
9.

A 13 yrs old girl presents with a painless dome shaped swelling of the ear lobule a few weeks after
piercing. which of the following is the most likely diagnosis?

a.
b.
c.
d.
e.

Malignancy
Keloid
Epidermal inclusion cyst
Hypertrophied scar
Abscess

Epidermal inclusion cyst is a flesh colored, dome-shaped nodule ranging from 0.5 to 5.0 cm in size, arising
from the infundibulum (upper portion) of a hair follicle and located in the dermis. Cheesy foul-smelling
keratinous content is found in the central part of the cyst, and is obtained on excision.

18

10. An 8 yrs old boy is brought to the dermatologist with a long history of severe sunburns and photophobia.
He has several lesions on the face that have recently changed in size and color alerting the dermatologist
to biopsy for possible malignancy. This condition is assosiated with Which of the following?

a.
b.
c.
d.

autoantibodies against hemidesmosomes


Inability to repair thymine dimers
the fragile X syndrome
Turners syndrome

Xeroderma pigmentosa is an autosomal recessive disorder characterised by Inability to repair thymidine


dimers that can form in the presence of uv light leading to Severe photosensitivity. The skin has multiple
freckles and hyperpigmented lesions and is prone to premature aging. The most important complication is
recurrent skin malignancy.
11. A 6 yrs old child recently came from a trip to the farm where he spent a lot of time playing with
animals. He has an itchy papular erythematous eruption spreading in a circular pattern over his leg with
central healing. Scrapings obtained from this lesion when examined under a microscope with KOH showed
septate hyphae. which of the following is the most likely diagnosis?

a.
b.
c.
d.

granuloma annulare
Tinea versicolor
candidal fungal infection
dermatophyte fungal infection

Epidermophyton,Trichophyton rubrum commonly cause the dermatophyte skin,scalp and nail infections.
Taenia corporis is the diagnosis in this patient.

19

12. A 2 year old child presents with fever, running nose and cough followed by a rash which appeared around
the ears then spread to the rest of his body. He has never been vaccinated since birth. You detect whitish
sand like spots on the buccal mucosa as shown in the picture below and a macular rash all over his body.
Most likely cause of his illness is

a.
b.
c.
d.
e.

Herpes simplex virus


Pox
Rubella virus
varicella Zoster virus
Rubeolla virust

The picture is showing koplik spots in the buccal mucosa which is present in children with measles caused
by rubeolla virus
13. A 6 months old girl is brought to the doctor with history of fever, irritability and skin rash for 6
days.Examination reveals erythema all over the body with the skin peeling off easily with gentle pressure
as shown below. Which of the following is the most likely cause?

a.
b.
c.
d.

microbial skin invasion


endotoxin mediated inflamatory response
mast cell degranulation
exotoxin mediated skin damage

scalded skin syndrome, the diagnosis in this patient is caused by staph or streptococcus infection resulting
in fever, rash and diarhea accompanied by skin exfoliation. The skin lesions are usually caused by exotoxin
mediated skin damage
14. A 70 yrs old male presents with several warty brown plaques on his face and back. the lesions appear
"stuck on" as if they can be peeled off. Which of the following is the correct diagnosis?

a.
b.
c.
d.

Seborheac keratosis
Atopic dermatitis
melanoma
Actinic keratosis

20

15. A 50 yrs old woman presents with a skin lesion on her upper eyelids as shown below. Biopsy shows
numerous lipid laden macrophages. This lesion can be seen in which of the following conditions?

a.
b.
c.
d.

liver cancer
Hyperthyroidism
Primary biliary cirhosis
Ulcerative colitis

Xanthelasmas are often seen in patients with Primary biliary cirhosis and are due to elevated lipid levels
16. A 30 yrs old lady presents with painful mouth ulcers and a skin rash. examination reveals erosions of the
buccal mucosa and blister like lesions over her trunk. The skin separates easily when gently rubbed
(Nikolsy's sign). autoantibodies directed against which of the following are most likely responsible?
a. Desmosomes
b. basement membrane
c. hemidesmosomes
d. tissue transglutein
Pemphigus is an autoimmune disease that results in painful flaccid bullae and erosions of the skin and
mucous membranes. Nikolsky sign is positive
17. A 30 yrs old female presents with skin lesions as shown in the picture. She is also suffering from
autoimmune thyroiditis. Which of the following is this condition assosiated with?

a.
b.
c.
d.

poor melanin synthesis


diminished pigment transfer to keratinocytes
absence of melanocytes in the skin
Melanosome aggregates within the cytoplasm of melanocytes

Vitiligo is an autoimmune disease characterised by the destruction and absence of melanocytes in the skin.
Other autoimmune diseases such as type I Diabetes mellitus and Hashimoto or Graves disease is often seen
in these patients
18. A 20 yrs old student presents with a sore throat and is prescribed medication for a week. She returns to
the doctor the same afternoon with a pruritic skin rash as shown below. This skin condition is
characterised by which of the following?

a.
b.
c.
d.

Acanthosis
Acantholysis
Epidermal spongiosis
Dermal edema

This skin condition is hives or urticaria and occurs due to the antigen(drug prescribed) induced
degranulation of focal mast cells through IgE sensitisation. this leads to microvascular hyperpermeability
21

and the formation of wheals due to dermal edema. It is a type I hypersentivity reaction.
19. A 55 yrs old woman presents with a hyperpigmented skin lesion on the face that looks like a huge mole.
She says the color of this mole has recently changed and shape has become different from what it was
before. A biopsy reveals the diagnosis of malignant melanoma. Which of the following parameters best
estimates her risk of developing metastasis?
a. Increase of melanosomes
b. active horizontal spread
c. active vertical growth
d. increased neutophil infiltration
e. Increased cellular atypia
Melanomas in the vertical growth phase contain cells that grow into the deeper dermal layers increasing
the risk of metastasis. Measurement of the depth of invasion (Breslow index) is the most prognostic
indicator.
20. A 50 yrs old patient presents with cough and shortness of breath for 3 months. On examination you
detect tender erythematous nodules over the legs as shown below. Cxray shows bilateral hilar opacities
and reticulonodular infiltrate. Most likely diagnosis is

a.
b.
c.
d.

necrobiosis lipoidica
erythema nodosum
erythema migrans
erythema multiforme

This vignette describes a patient with sarcoidosis and erythema nodosum is the skin manifestation that
frequently occurs in this condition
21. A 6 yrs old child is brought to your clinic with severe pruritic lesions on axillary fold, genitals and
interdigital spaces on the hands with linear burrows in the affected areas. Her mother says her younger
son is also having the same problem. Most likely diagnosis is
a. scabies
b. dermatophyte fungal infection
c. atopic dermatitis
d. celiac disease
22. A 30 yrs old male patient presents with weakness, myalgia, and knee joint pain. His wife says they
recently went camping to a forest where they were bitten by multiple ticks. On examination you detect a
bulls eye type of erythematous skin lesion on his legs. His condition is most likely caused by which of the
following organisms?

a.
b.
c.
d.

Ricketsia ricketsii
Streptococcus pyogenes
Yersinia pestis
Borellia burgdorferi

22

This is a vignette describing Lyme disease which is spread by tick bites. The causative organism is a
spirochaete Borrelia burgdoferi and the skin lesion is called erythema migrans
23. A 5 yrs old girl is brought by her mother for treatment of her skin condition. On examination you detect
honey coloured crusted lesions at the angle of her mouth as shown in the picture. grams stain of pus
obtained shows gram positive coccii. Which of the following is the most likely diagnosis?

a.
b.
c.
d.

Impetigo
acne vulgaris
folliculitis
cellulitis

24. A 50 yrs old patient is seen by the dermatologist with nodular skin lesions as shown below. Which of the
following is the most likely diagnosis?
a. Neurofibromatosis
b. Sqaumous cell carcinoma
c. Bullous pemphigoid
d. Seborheac keratosis
25. A 20 yrs old college student is seen on a routine physical exam to have multiple waxy papular swellings
around the eyes and elbows. Skin around his neck appears like that of a plucked chicken. His serum
cholesterol levels are normal. Which of the following is the most likely diagnosis?
a. Rheumatoid arthritis
b. Familial hyperlipidemia
c. Neurofibromatosis
d. pseudoxanthoma elasticum
26. A 2 yrs old boy is brought to the pediatrician with the mother complaining of the child's short stature.
You detect the following on examination. Macrocephaly,frontal bossing, short arm and legs as well as
trident hand. Which of the following is the most likely diagnosis?
a. Tay Sachs disease
b. Achondroplasia
c. Cystic fibrosis
d. Prader willi syndrome
Achondroplasia is caused by mutation of FGFR 3 gene leading to inhibition of cartilage formation
27. A 30 yrs old school teacher presents with severe pain and stiffness of his back for 12 months. He is
unable to tie his shoelaces. Physical examination reveals limited anterior flexion of the spine. Xray spine
shows Syndesmophytes bridging multiple vertebrae resulting in spinal fusion. Which of the following is
most strongly assosiated with this patient?
a. Immunoglobulin defeciency
b. Specific HLA class I serotype
c. Specific HLA class II serotype
d. deficient complement component
This vignette describes a patient with Ankylosing spondylitis. This is strongly assosiated with HLA class I
(HLA B27) antigen.

23

28. A 45 yrs old school teacher presents with retrosternal burning and difficulty in swallowing her meals.
She also says her fingers turn white and than blue when she is out in the cold weather. You detect thick
shiny skin on her fingers and painful subcutaneous nodes. Which of the following antibodies is most likely
to be present in this patient?
a. anti SSA & SSB
b. anti ds DNA
c. anti centromere
d. anti histone
This vignette describes the CREST syndrome with Calcinosis, Raynauds phenomenon, esophageal
dysmotility, sclerodactyl and telengactasia. Anti Centromere antibodies are present.
29. A 34 yrs old woman presents with fever, chest pain, shortness of breath, wt loss, fatigue, malaise, and
swollen, tender joints for 6months. She denies use of any medication. P/E is significant for a
maculopapular eruption over sun exposed areas, including the face with photosensitivity. Peripheral blood
smear reveals anemia, leucopenia, thrombocytopenia. C x-ray: bilateral pleural effusion. Which of the
following antibodies will be specificaly seen in this disease?
a. anti centromere
b. antinuclear
c. anti SSA & SSB
d. anti ds DNA
e. anti histone
This vignette describes a patient with SLE and both ANA and anti ds DNA antibodies will be present.
However anti ds DNA is specific for SLE. If it is drug induced anti histone antibodies will be present.
30. A 40 yrs female presents with bilateral cheek swelling for 4 months. she also complains of dry mouth
and ulcers in the oral cavity as well as eye irritation and multiple joint pain. On examination you detect
dry conjunctiva and cornea, oral ulcers and bilateral parotid swellings. She was diagnosed with RA 2 yrs
ago. which of the following is the most likely diagnois?
a. Reiters syndrome
b. Sjogrens syndrome
c. SLE
d. Antiphospholipid syndrome
pts w sjogrens syndrome have a high predisposition to B cell lymphoma & have anti SSA & SSB Abs circulating
31. A 45 yrs old lady presents with weakness of 6 months duration. On examination you detect proximal
muscle weakness and a rash as shown below. which of the following is the most likely diagnosis?

a.
b.
c.
d.
e.

Polymyositis
Polymalgia rheumatica
Dermatomyositis
Psoriasis
Rheumatic fever

Dermatomyositis is characterised by proximal muscle weakness, a heliotrope skin rash around the eyes
and gottrons papules on the knuckles

24

32. A 50 yrs old lady presents with generalized pain of her muscles, fatigue and restless nights. she gets up in
the morning feeling she has not slept at all and has emotional disturbances and poor memory. physical
exam reveals significant tenderness when her muscles are palpated and her reactions are wildly out of
proportion to touch. Which of the following is the most likely diagnosis?
a. Fibromyalgia
b. Polymyalgia rheumatica
c. Polymyositis
d. Myasthenia gravis
33. A 70 yrs old woman presents with pain, weakness and stiffness of her shoulders and hip muscles for 5
months. She was diagnosed with Temporal arteritis 2 yrs ago. which of the following is the most likely
diagnosis?
a. Polymyositis
b. dermatomyositis
c. Fibromyalgia
d. Polymyalgia rheumatica
PMR is the syndrome frequently seen in patients with Temporal giant cell arteritis. It presents as
indicated in this vignette and patients have a high ESR
34. A 26 yrs old patient with HLA-B27 antigen presents with a triad of arthritis, conjunctivitis, and
urethritis. He also has mouth ulcers. His Urethral smear may be positive for
a. gonorhea
b. Ecoli
c. chlamydia
d. gram negative diplococci
This patient has Reiters syndrome characterised by Chlamydia urethritis, reactive arthritis, conjunctivitis.
These patients often have HLA B27 haplotype.
35. A 28 year old sexually active adult comes with history of fever of 5 days duration associated with
swelling of the left knee joint. No history of fall or trauma. The patient was diagnosed with Rheumatic
fever at the age of 12 years. His BP is 130/70, pulse 90/min. The joint is swollen, warm and tender to
touch. Range of motion is restricted and there is no history of early morning stiffness. Joint aspiration
shows 16,000 WBCs predominantly polymorphonuclear neutrophils. Which of the following is the most
likely diagnosis?
a. osteoarthritis
b. Rheumatoid arthritis
c. septic arthritis
d. reactive arthritis
36. Foot examination is performed on a 75-year-old man who had been diagnosed with Type 1 diabetes
mellitus at age 15. While the man has no complaints of foot pain, both feet appear clearly deformed,
with loss of the normal arches and abnormal orientation of the toes. No inflammation is noted. X-ray
films show deformed bones with new bone formation adjacent to the bone cortex. Several large, bizarre
shaped osteophytes are seen at the joint margins. Which of the following is the most likely diagnosis?
a. rheumatoid arthritis
b. osteoarthritis
c. Gout
d. Pseudogout

25

37. A 69-year-old retired police officer comes to the emergency department complaining of Excruciating
right foot pain. The pain began approximately 6 hours ago, immediately on waking. One night earlier, he
had gone out for a steak dinner and drank half of a bottle of wine to celebrate his recent retirement. His
past medical history is significant for hypertension, for which he takes hydrochlorothiazide. On physical
examination, he is afebrile but appears acutely uncomfortable. There is swelling and tenderness in the
right ankle and in the first right toe. Laboratory tests reveal a white blood cell count of 12,400/mm3
and a hematocrit of 39%. Serum electrolytes and liver function tests are normal. Uric acid is 4.1 mg/dL
(normal 2.5-5.5mg/dL). Which of the following will most likely be seen on analysis of the joint fluid
obtained by arthrocentesis?
a. Negatively birefringent needle-shaped crystals
b. Negatively birefringent rhomboid crystals
c. Gram-negative rods
d. Gram-positive cocci in pairs and chains
38. A 5-year-old girl is brought to the clinic with a 3-day history of fever, dyspnea, and intermittent joint
pain. She is generally healthy but according to her mother, she had a 'cold and sore throat' about 1 month
ago. On physical examination, her temperature is 39.6 C (103.2 F), blood pressure is 94/60 mm Hg, pulse
is 114/min, and respirations are 22/min. Her knees and elbow joints are swollen and tender to
palpation. There is a grade III/VI diastolic murmur best heard at the apex. Multiple fine, pink macules are
noted on her trunk. These macules are blanching in the middle. Which of the following is the most likely
diagnosis?
a. septic arthritis
b. Lyme's disease
c. rheumatic fever
d. juvenile rheumatoid arthritis
39. A 30-year-old woman comes to the physician with low back pain and a swollen and painful right ankle
and left knee for 1 week. She also reports temperatures to 39.0 C (102.2 F). She recalls that a few
weeks ago, she had diarrhoea, abdominal cramps, and fever, which lasted for a few days. She does not
have previous history of joint diseases or sexually transmitted diseases. Examination reveals skin pustules
and crusted vesicles on her palms and soles, red conjunctivae, and arthritis of the left knee and right
ankle. There is no hepatomegaly, Spleenomegaly, or lymphadenopathy. Laboratory analysis show an
elevated erythrocyte sedimentation rate (ESR), but no anaemia. Which of the following is the most likely
diagnosis?
a. rheumatoid arthritis
b. psoriatic arthritis
c. reiters syndrome
d. gonococcal arthritis
40. A 17-year-old boy is brought to the clinic because of chronic low back pain for the past 8 months. He
was the most promising member of the high school swim team but was forced to quit because of his back
pain. The pain begins frequently at night, radiates down the thighs, and is accompanied by pronounced
stiffness of the lumbar spine. He denies any gastrointestinal or genital infections. His temperature is 37.0
C (98.6 F). Examination reveals moderate limitation of back motion and tenderness of the lower spine. A
diastolic murmur along the left sternal border is heard on chest examination. Laboratory studies show an
elevated erythrocyte sedimentation rate (ESR) and negative rheumatoid factor. X-ray films of the
vertebral column and pelvic region show flattening of the lumbar curve and subchondral bone erosion
involving the sacroiliac joints. Which of the following is the most likely diagnosis?
a. Degenerative joint disease
b. Seronegative rheumatoid arthritis
c. Reiter syndrome
d. Ankylosing spondylitis no ans on qstream- this had 80% chosen

26

41. A 53-year-old man comes to the office complaining of severe right toe pain. The patient was awakened
in the middle of the night by a sharp, burning pain localized to the first Metatarsophalangeal joint. It is
noted to be warm, painful to touch, and oedematous; the patient cannot move the joint in any direction
without extreme pain and will not allow the toe to be covered. The patient's medical history is significant
for hypertension for which he takes a Thiazide diuretic. He consumes about 3 pints of beer per day.
Which of the following is most likely to confirm the diagnosis in this patient?
a. Determination of serum urate concentrations
b. Aspiration of fluid from the affected joint
c. Radiologic examination of the affected joint
d. Determination of urine urate concentrations
42. 40-year-old woman has an 8-month history of diffuse joint pain and swelling that involves both hands
and knees. She states that she cannot get any work done in the morning due to the pain, but it usually
subsides as the day progresses. She tires easily and constantly feels "feverish". Her temperature is 37.8 C
(100 F), blood pressure is 110/70 mm Hg, pulse is 60/min, and respirations are 18/min. Physical
examination shows tender, swollen, and "boggy" hands and knees. Laboratory studies show:
Hematocrit............34%
Haemoglobin...........10 g/dL
A radiograph of this patient's knee is most likely to show which of the following?
a. Subchondral tophi
b. bone erosions
c. Subchondral cyst formation
d. Osteophyte formation
43. A 9yrs old child with h/o easy bruising and hyperextensible joints presents to ER after dislocating his
shoulder for 5th time this yr. Which of the following is the most likely diagnosis?
a. Ehlers Danlos syndrome
b. Pseudoxanthoma elasticum
c. Achondroplasia
d. Osteogenesis imperfecta

27

ICM 1 and ICM 2 : Dermatology Qstream:


1.

A 50 yrs old man presents to his primary care physician with a pearly nodule on his face. This nodule is
heaped up, has translucent borders and telangiactasia surrounding the lesion. Microscopy of a punch
biopsy reveals nests of cells with a pallisading growth pattern. Based on this description and the image
below what is the most likely diagnosis?

a.
b.
c.
d.
e.
2.

A 6 yrs old child recently came from a trip to the farm where he spent a lot of time playing with
animals. He has an itchy papular erythematous eruption spreading in a circular pattern over his leg.
Scrapings obtained from this lesion when examined under a microscope with KOH showed septate hyphae.
Most likely diagnosis is

a.
b.
c.
d.
3.

Seborrhaec dermatitis
Squmous cell carcinoma
Basal cell carcinoma
Malignant melanoma
Psoriasis

Scabies
Candidal fungal infection
Taenia versicolor
Granuloma annulare

A 2 year old child presents with fever, running nose and cough followed by a rash which appeared around
the ears then spread to the rest of his body. He has never been vaccinated since birth. You detect whitish
sand like spots on the buccal mucosa as shown in the picture below and a macular rash all over his body.
Most likely cause of his illness is

a.
b.
c.
d.
e.

Allergic rash
Rubella virus
Eczema
Varicella Zoster virus
Rubeolla virus

28

4.

Two days after going on a hike in the forest, a 12 yrs old girl develops erythematous plaques and vesicles
on her legs as shown below. Some plaques appear to be developing crusts. She also complains that the
lesions are severely pruritic. What is the most likely diagnosis?

a.
b.
c.
d.
e.

Contact dermatitis
Atopic dermatitis
Lyme disease
Actinic keratosis
Impetigo

29

ICM 1 & 2 : Connective Tissue Disorders Qstream:


1.

A 60 yrs old lawyer presents with severe headache, pain and stiffness of his shoulder muscles for 6
months. He finds it difficult to comb his hair and 2 months ago had a very tender cord like swelling in his
right temporal region. His ESR was found to be raised when his blood work was done at that time. His
present symptoms indicate a most likely diagnosis of
a. Dermatomyositis
b. Myaesthenia Gravis
c. Polymyositis
d. Polymyalgia Rheumatica
e. Fibromyalgia

2.

A 35 yrs old woman presents at the ER with chest pain, breathlessness, cough, fever and joint pains for
2 months. She is a non smoker and does not drink alcohol. She has lost 20 pounds recently without any
dietary changes. Past history is significant for recurrent abortions. On examination you detect a malar
skin rash and chest examination reveals dull percussion note, reduced TVF and absent breath sounds on
both sides. Lab test results reveal the following: CBC results: anemia, leucopenia, thrombocytopenia and
raised ESR, Urinalysis: heavy proteinurea. Chest Xray shows a pleural effusion. Most likely diagnosis is
a. Rheumatoid Arthritis
b. Lung cancer
c. SLE
d. Tuberculosis
e. Sarcoidosis

3.

A 30 year old woman visits her family physician complaining of weakness in her arms and legs. She has
had difficulty getting up after sitting in her chair and has noticed this for about 2 weeks now. She has
also noticed the appearance of a rash on her face. Physical examination reveals the presence of a
heliotrope rash on her face and papules over her knuckles as shown below. What is the most likely
diagnosis?

a.
b.
c.
d.
e.

Myotonic dystrophy
Polymyositis
SLE
Dermatomyositis
Scleroderma

30

ICM 2 : Respiratory System Qstream:


1.

A 68 year old female presents to her physician with a chronic cough with hemoptysis and weight loss.
Before she stopped smoking 8 years ago, she used to smoke nearly 2 packs per day for over 20 years.
Physical exam reveals wheezes. The wheezes do not respond to a nebulized beta agonist treatment given
in the office. A chest xray reveals a central mass with cavitation. Serum calcium is elevated. What is the
most likely diagnosis?
a. squamous cell carcinoma of the lung
b. mesothelioma
c. bronchial carcinoid tumor
d. adenocarcinoma of the lung
e. small cell carcinoma of the lung
The combination of chronic cough with hemoptysis and weight loss suggests malignancy. Squamous cell
carcinoma shows up as a central mass with cavitation on xray. It makes a PTH like peptide that leads to
hypercalcemia.

2.

A 54 year old male has been diagnosed with lung cancer. He presents to you because he is having
problems breathing when he lays down at night. He also complains of headache and his family often
asks if hes feeling warm because they said his face is flushed. He has noticed that he can no longer
button the top button of his dress shirts though he denies weight gain. What is the reason for these
symptoms?
a. compression of the superior vena cava
b. compression of the recurrent laryngeal nerve
c. compression of the brachial plexus
d. secretion of serotonin
This is superior vena cava syndrome due to compression of the vessel

3.

A 23 year old male presents with dyspnea and SOB following a motor vehicle crash. He has no significant
past medical history. On examination he has a pulse of 117, a blood pressure of 92/64 and the pulse
oximeter gives you a reading of 89%. He has swelling and obvious deformity to the right side of his
chest. He has tenderness with palpation of his ribs and absent breath sounds on the right side. You give
supplemental oxygen and note no change in his oxygen saturation. A chest xray shows tracheal deviation
to the left and minimal vascular markings on the right. To prevent cardiovascular collapse
a. introduce a needle into the 8th intercostal space, mid-axillary line, along the superior border of
the inferior rib
b. introduce a needle into the 2nd intercostal space, mid-clavicular line, along the inferior border of
the superior rib
c. introduce a needle into the 8th intercostal space, mid-axillary line, along the inferior border of
the superior rib
d. introduce a needle into the 2nd intercostal space, mid-clavicular line, along the superior border
of the inferior rib
This patient has a tension pneumothorax due to his fractured ribs. You need to release the tension caused
by the free air in the lung

4.

A 4 year old female is brought the ED by her father. She is in moderate respiratory distress. She is
tachycardic and tachypneic and is begans to make a high pitched noice with inspiration when she gets
agitated. He states she has had viral symptoms for the past 2 days but tonight she began with deep
cough that sounded like a barking seal. What is the most likely cause of this illness?
a. respiratory syncytial virus
b. haemophilus influenzae type B
c. parainfluenza virus
d. adenovirus
e. influenza A virus
This is the classic presentation of croup. It is caused by the parainfluenza virus

31

5.

A 23 year old woman presents with new onset wheezing. She denies a history of atopy in herself or
other family members. She had started having headaches in the past few weeks for which she has been
taking increasing amounts of aspirin. On exam she is breathless and using accessory muscles. What else
might you notice on physical examination?
a. darkening underneath the eyes bilaterally
she has intrinsic asthma
b. flatness to percussion over the lungs
c. nasal polyps
d. a pleural rub
e. excoriations in the flexural creases
Wheezing and aspirin sensitivity is associated with nasal polyps. A patient with this picture and no
history of allergic disease may be in the small percentage of people with intrinsic, or non-allergic, asthma.

6.

A couple whose infant son has just been diagnosed with cystic fibrosis comes in to speak with you about
the complications of the disease. Which of the following statements does not characterize cystic fibrosis
long term problems?
a. Vitamin and nutritional deficiencies lead to failure to thrive and poor growth.
b. Derangements in ion transport leads to the proliferation of pancreatic tissue.
c. The lungs have an impaired ability to clear mucus which leads to recurrent bacterial infections.
d. There is low fertility in males due to blockage of the vas deferens.
e. Excessive reabsorption of liquids in the GI tract leads to dehydrated intraluminal contents and
bowel obstruction.
Pancreatic tissue is destroyed over time.

7.

A 7 year old female with cystic fibrosis presents to the emergency department with weakness and
confusion following playing in a soccer game. Her blood pressure is 60/30 and her pulse is 130. She has
dry oral mucosa. What is the reason for her condition?
a. free water loss with sweating
b. sodium loss with sweating
c. chloride loss with urine
She secretes a high sodium high chloride sweat. Due to impaired ion transport, she cannot resorb sodium
and chloride.

8.

In silicosis, the number of alveolar macrophages are diminished. What types of infections are these
patients at high risk for later in life?
a. encapsulated bacteria
b. mycobacterial
c. pseudomonal
d. mycoplasmal
e. candidal

9.

A 62 year old miner with known silicosis presents with a several week history of worsening cough,
weight loss and night sweats. A PPD is positive, and he is subsequently diagnosed with pulmonary
tuberculosis. What is the explanation for this?
a. Impaired TH2 cell function
b. Suppressed antibody response
c. Impaired killing by macrophages
d. Ineffective mucociliary clearance
e. Extensive pulmonary fibrosis
Silica is toxic to alveolar macrophages which is the bodys primary defense when it comes to
mycobacterial infection. In silicosis, there are fewer of these cells around, so TB is able to flourish.

32

10. A 15 year old high school student presents with a three week history of a dry, persistent cough and
malaise. He denies history of fever or chills, body aches, chest pain, exposure to farm animals or recent
travel. The principal of the school states that a few other students have also been feeling the same
way. A chest x-ray reveals diffuse, bilateral patchy infiltrates. Which of the following is the most
likely cause of this patients condition?
a. Mycoplasma pneumoniae
b. Coxiella burnetii
c. Klebsiella pneumonia
d. Streptococcus pneumonia
e. Legionella pneumophila
Mycoplasma pneumonia commonly presents with these atypical symptoms of pneumonia and can occur
in institutions where younger people often work, study or reside.
11. A 32 year old African-American female with a malar rash comes to the office with complaints of
worsening dyspnea over the past couple of weeks. She denies fever, cough or other ill symptoms. She
does states she feels more fatigued, in general. On physical exam you note diminished breath sounds at
the bases, especially on the right side, with flatness on percussion. In both phases of respiration you note
a slight creaking sound at the right base, not a crackle or a wheeze. What is the most likely diagnosis?
a. asthma
b. atelectasis
c. pneumothorax
d. pleural effusion
e. pneumonia
The creaking sound is a pleural rub, and these findings are otherwise consistent with a pleural
effusion. The patient has lupus which often has a pulmonary component.
12. Which of the following vignettes apply to the chest xray pictured below?

a.
b.
c.
d.
e.

This patient has complaints of difficulty breathing following a car accident and several rib
fractures.
This patient is a healthy female and runs long distances on a regular basis.
This patient has a restrictive lung disease and gets pulmonary infections frequently.
This patient has a 40 pack-year smoking history. He has episodes of dyspnea and increased
sputum production which are relieved with bronchodilators and anticholinergics.
This patient presents with a 4 day history of cough and fever, with brownish sputum and pain
with inspiration.

This chest xray shows hyperinflation and flattened diaphragms which are characteristic of obstructive
lung disease.

33

13. A 10 year old boy with cystic fibrosis presents to his pediatrician with a history of fever to 101F and a
cough productive of blue-green tinged sputum for the past 3 days. On PE there are crackles in the right
middle lobe, and CXR shows a right middle lobe infiltrate. Which of the following is the most likely
pathogen?
a. Streptococcus pneumonia
b. Mycoplasma pneumonia
c. Klebsiella pneumonia
d. Pneumocystis jiroveci
e. Pseudomonas aeruginosa
Pseudomas aeruginosa infections are common in CF patients. These bacteria thrive in the viscous mucous
layer of the lungs. It is also known to produce a blue-green pigment called pyocyanin. Streptococcus is a
common cause of community acquired pneumonia in healthy patients. Klebsiella is more common in
patients who are hospitalized. Pneumocystic jiroveci is an opportunistic pathogen seen primarily in
HIV/AIDS patients. Mycoplasma pneumoniae has a more insidious presentation.
14. A 63 year old retired miner presents with progressive, worsening dyspnea. His PMH is significant for
hypertension and a 30 pack-year smoking history. He smells of tobacco in your office today. On PE you
note bilateral end inspiratory crackles and digital clubbing. His chest xray shows ferruginous
bodies. Which of the following is true regarding this disease process?
a. Cancer of the lung pleura has a strong association with this disease.
b. Lung reduction surgery can improve his symptoms and change the course of the disease process.
c. This is generally a rapidly progressing disease, with less than 5 years between exposure, start of
symptoms and death due to respiratory failure.
d. Removing this patient from the exposure will cure his symptoms.
e. A PPD on this patient would likely be negative due to anergy.
As a miner, he has been exposed to inorganic dust particles. His chest CXR shows ferruginous bodies
which are found in asbestosis. 80% of mesotheliomas, cancer of the lung pleura, are associated with
asbestosis exposure. Asbestosis has a latency period of 15-40 years. Lung reduction surgery may be
helpful in emphysema. Pneumoconiosis patients are not anergic. There is a tissue reaction to the presence
of the inorganic dust fibers. They are visible in the lungs and hard to clear. Removing him from the mine
will not help him at this point.
15. A 39 year old man presents to the Emergency Department. Through a translator he gives a history of
fever, night sweats, weight loss, and hemoptysis. CXR shows patchy upper-lobe infiltrates with
cavitation. Which of the following is true with regard to this disease?
a. Its ability to retain the Ziehl-Neelsen stain following decoloration with acid-alcohol is highly
suggestive for the disease.
b. Chocolate agar is the preferred medium to grow this pathogen.
c. This disease causes non-caseating granulomas and massive pulmonary fibrosis.
d. His symptoms are most likely due to a primary infection of the pathogen.
e. A PPD placed on this patient is likely to be negative.
This patient has tuberculosis. Its ability to retain the Ziehl-Neelsen stain is what makes it an acid-fast
bacillus. Due to placement of the pulmonary lesions, this is likely a reactivation or secondary infection
rather than a primary infection. TB causes caseating granulomas. PPD indicates exposure and will likely
be positive. Chocolate agar facilitates the growth of fastidious organisms such as Haemophilus and
Neisseria, not TB.

34

16. A 61 year old female comes in with right shoulder pain. She has also had fatigue and an unexplained
weight loss. You suspect malignancy, and a chest xray reveals a mass near the pleural apex, in the 2nd
intercostal space, lateral to the manubrium of the sternum. What other finding would you expect due to
local tumor extension?
a. anosmia
b. leucocoria
c. bitemporal hemianopsia
d. unilateral deafness
e. ptosis
The malignancy is a Pancoast tumor and gives rise to Horner syndrome: anhydrosis, miosis, ptosis.
17. You are working in the emergency department when a mother brings her 5 year old son in. He is having
difficulty breathing, and he is drooling. The mother states he started having fever yesterday but denies a
cough or any other ill symptoms. Today he refuses to eat or drink anything. On exam, he is febrile. He is
leaning forward and looks anxious, but he is not crying and will not speak. On laparoscopy a cherry red
epiglottis is noted. A complete blood count shows a white count of 23,000 with a neutrophil
predominance. Which of the following statements is true regarding his condition?
a. There is a characteristic family history associated with his symptoms.
b. A barky, brassy cough worse at night is common with this viral condition.
c. This is likely due to a hymenoptera(bee/wasp) allergy.
d. A review of his immunization status will reveal some missed vaccinations.
e. Further questioning will likely reveal a history of recent travel outside the country.
This is the classic presentation of epiglottitis. It is a bacterial infection caused mostly by Haemophilus
Influenzae type B. With vaccination of children, the disease has dropped dramatically. With the presence
of fever from the day before and elevated WBC, allergic reaction is unlikely. Croup is characterized by a
barky cough caused by the parainfluenzae virus. Travel history has no bearing on this disease.
18. A 59 year old woman presents with progressive shortness of breath that is affecting her activities. She
has a 1 ppd smoking history for the past 20 years. On exam she has diminished breath sounds
bilaterally and scattered wheezes. Which of the following is likely to be increased?
a. forced vital capacity
b. functional residual capacity
c. elastic lung recoil
d. expiratory flow rates
19. A patient with asthma has recurrent pulmonary infiltrates. The CBC shows eosinophilia. There is
bronchiectasis on chest CT. Which of the following is he likely to be colonized with?
a. aspergillus fumigatus
b. adenovirus
c. S pneumonia
d. Legionella
e. P aeruginosa
20. Serum from pt A impairs elastin degradation by neutrophil products in vitro whereas serum from patient
B does not have that ability. Pt B should be strongly warned to avoid:
a. Iron containing pills
b. direct sunlight exposure
c. high fat diet
d. smoking
e. Food containing phenylalanine
Alpha 1 antiptrypsin deficiency. Cigarette smoking puts him at very high risk. Also associated
with emphysema and liver cirrhosis.

35

21. A 6 year old boy is sitting in class swishing a pea-sized pencil eraser around in his mouth. He is
accidentally jostled by a classmate and the boy aspirates the eraser. Which is the most likely area of the
lung for the foreign body to settle?
a. Right mainstem bronchus, lower portion of the inferior lobe
b. Right mainstem bronchus, superior portion of the inferior lobe
c. Left mainstem bronchus, lower portion of the inferior lobe
d. Left mainstem bronchus, superior portion of the inferior lobe
Of the bronchi, the right main stem bronchus is more common because it is wider and more vertical than
the left. It goes into the inferior lobe. If the person is sitting upright, it goes into the lower portion of the
inferior lobe. If the person is lying supine when they aspirate, it goes into the upper (superior) portion of
the inferior lobe.
22. A mother brings her 3 month old infant in because shes concerned that he had a fever last night and he
appears to be breathing harder than usual today. On exam you see an infant with an increased
respiratory rate, intercostal retractions, audible nasal congestion and a cough. You hear wheezing on
auscultation of the lungs. Which of the following statements is true about this disease?
a. This disease does not respond well to nebulized albuterol therapy.
b. Conjunctivitis is commonly a part of the presentation of this disease.
c. Staccato coughing with a dramatic inhalation at the end characterizes this disease in older
children.
d. The most common etiology of this disease is parainfluenza.
This is a typical presentation of bronchiolitis in a younger child which is caused most commonly by
respiratory syncytial virus. Though wheezing is characteristic, it is the copious secretions that cause the
airway narrowing so bronchodilator therapy is not usually very effective. Parainfluenza is a common
cause of croup. Conjunctivits is commonly seen with adenovirus infection, and staccato coughing with a
dramatic inhalation at the end is the presentation for whooping cough or pertussis.
23. Different factors determine hemoglobins affinity for O2. Which of the following factors is associated
with an increased hemoglobin affinity for O2?
a. high carbon monoxide levels
b. high altitude
c. high levels of 2,3 diphosphoglycerate
d. acidotic blood pH
Carbon monoxide binds oxygen with a higher affinity than adult hemoglobin does. It will shift the
hemoglobin-oxygen dissociation curve to the left. Other scenarios that are associated with increased
hemoglobin affinity are: low/colder body temperatures, alkalotic blood pH, and presence of fetal
hemoglobin. The other scenarios are associated with a decreased affinity for hemoglobin and will shift
the curve to the right.

36

ICMII Cardiovascular Qstream:


1.

A big baby(5kg) born of a diabetic mother, is found to be breathless, tachypnec and cyanotic right from
birth. A systolic murmur is present on auscultation of the heart. Prostaglandin E is given as the child is
prepared for surgery. A defect in Which of the following embryological processes resulted in this
condition?
a. fusion
b. looping
c. spiralling of A/P septum
d. obliteration
This is a case of Transposition of the Great arteries and is due to a spiralling defect of the Aorticpulmonary(A/P) septum that divides the Trunkus arteriosus.

2.

A 2 yrs old child presents with spells of dyspnea, cyanosis, and squatting after play. O/E, you detect
clubbing, lt parasternal heave and an ejection systolic murmur in the pulmonic area. Xray chest reveals a
boot shaped heart. Which of the following is he also likely to have?
a. severity of pulmonic stenosis
b. Age of the child
c. size of the over riding Aorta
d. decreased pulmonary blood flow
This is a case of Fallot's tetralogy and the severity of symptoms is determined by the degree of pulmonic
stenosis. Pulmonary blood flow and vascularity is diminished in these patients

3.

A 16 yrs old girl presents with failure to begin her menstrual periods and past history of frequent severe
headaches and nosebleeds. On examination her Bp is 180/120 in the upper extremity and 100/60 in
the lower. pulsatile vessels are seen in the neck and palpated along the ribs. She has a webshaped neck
and increased carrying angle of the elbow. Xray shows notching of ribs. Which of the following is the
most likely diagnosis?
a. Pregnancy
b. Cushings syndrome
c. Conns syndrome
d. Vascular ring
e. coarctation of Aorta
This patient has the clinical features of Turners syndrome with Coarctation of the Aorta resulting in high
blood pressure in the upper extremity. A bicuspid aortic valve is also commonly seen in assosiation with
Turners syndrome

4.

An 80 yrs old woman presents with scalp pain and jaw claudication of 6 weeks duration. On
examination you detect a tender, cord like artery in the temple region. Lab work reveals a raised ESR.
Which of the following conditions is she likely to develop with her present disease?
a. Fibromyalgia
b. Polymyalgia rheumatica
c. SLE
d. Myotonic dystrophy
This vignette describes Giant cell arteritis which is a granulomatous arteritis involving large
arteries.Takayasu arteritis has a similar pathological feature. Patients with Giant cell arteritis may
develop the syndrome of Polymyalgia rheumatica characterised by pain and stiffness involving the
muscles of the shoulder girdle. Most of these patients have a high ESR

37

5.

A 30 yrs old female presents with fever, weight loss, joint pains, lower limb claudication and myalgia.
You detect absent/weak carotid/radial pulses but femoral pulse is normal. Which of the following is the
most likely diagnosis?
a. Takayasu arteritis
b. PMR
c. Fibromyalgia
d. Kawasaki disease
e. Polyarteritis nodosa
Takayasu arteritis is a vasculitis involving large arteries, usually one of the proximal arteries arising from
the aorta and results in fibrosis and hence the weaker pulses felt. The vascular insufficiency that occurs
causes claudication

6.

A 40 yrs old lady presents with fever and weight loss of 10 pounds in 4 months.She has a past history
of hep B and multiple skin lesions suggestive of ischemia and gangrene. biopsy reveals a necrotising
vasculits involving medium sized arteries. Which of the following is the most likely diagnosis?
a. Beurgers disease
b. Liver cancer
c. Liver Cirhosis
d. Polyarteritis nodosa
e. Henoch Schonlein Purpura
@ PAN is a segmental, transmural necrotizing inflammation of medium to small sized arteries. Vessels of
the kidneys, Liver, GIT and skin are commonly involved resulting in ischemia, infarction or hemorrhage.
Lungs are rarely involved.

7.

An 8 yrs old boy is brought to hospital with an acute onset of skin rash, abdo pain and swollen joints. He
dies in hospital a week later from renal failure. o/e you detect palpable reddish purple skin lesions on his
legs.Which of the following will you find on biopsy of kidney and skin vessels?
a. C- ANCA
b. Ig A deposits
c. Antibodies against glomerular basement membrane
d. Ig G deposits
This is a case of Henoch Schonlen Purpura(Leukocytoclastic vasculitis) presenting with purpura and renal
involvement. It is characterised by Ig A deposits in the vessels of the skin and kidneys

8.

A 40 yrs old male patient presents with recurrent attacks of sinusitis. one week back he had a bout of
cough with hemoptysis. he also recalls an episode of haematuria one month ago. on examination you
detect nasal ulceration. lab work shows RBC casts in urine. xray chest: lung nodules, biopsy reveals
granulomatous vasculitis of small arteries. Which of the following is the most likely diagnosis?
a. Good pastures syndrome
b. Polyarteritis nodosa
c. Churg strauss syndrome
d. Henoch Schonlein Purpura
e. Wegeners Granulomatosis
This is a typical presentation of Wegeners granulomatosis and is characterised by the presence of cANCA antibodies against neutrophils

38

9.

A 20 yrs old marathon runner has a mean arterial pressure of 95 mm hg at rest. His pulse is 50bpm and
a systolic murmur is detected on auscultation. After a one hr race his MAP has risen to 105. A decrease
in which of the following during his running most likely accounts for the observed finding?
a. renal blood flow
b. Systemic vascular resistance
c. systolic pressure
d. stroke volume
The mean arterial pressure remains fairly stable during exercise due to an adaptive decrease in the
systemic vascular resistance. Exercising muscle releases vasodilatory substances such as adenosine that
causes the decrease in the systemic vascular resistance whilst allowing more blood flow to the exercising
muscle.

10. A 75 yrs old man presents with headache and light headedness. On examination he is noted to have a
Blood pressure of 180/70 and heart rate of 75 beats per minute.Which of the following is the most
likely cause of his hypertension?
a. Increase in sympathetic tone
b. Renal artery stenosis
c. Aortic stiffening
d. Decrease in cardiac output
e. Decrease in lung capacity
Isolated hypertension is caused by age related decrease in the compliance (increased stiffening) of the
Aorta and its proximal major branches. Renal artery stenosis and increase in sympathetic tone would
result in both systolic and diastolic elevation of blood pressure.
11. A 60 yrs old female is seen in the ER with severe chest pain of 2 hrs duration. An ECG recording
demonstrates a tachycardia with wide QRS complexes but no P waves. Which of the following is the
most likely cardiac pacemaker location in this patient?
a. Atria
b. AV node
c. Ventricles
d. SA node
An ECG can identify the location of an abnormal pacemaker. If the pacemaker was the SA node the rate
will be normal around 70 and the P wave would be present. If the pacemaker is the AV node the rate is
slower(below 60) and P waves are absent with normal QRS(narrow complexes).this happens if there is
impaired conduction between the SA and AV node and the AVnode takes over as the pacemaker. Wide
QRS complexes indicate a ventricular rythm in this case Ventricular tachycardia
12. A 25 yrs old female patient presents with a 6 month history of Paroxysms of severe headache
associated with palpitations, sweating and anxiety. Bp is 200/120 and you note it to be 140/100, 3
hrs later. Which of the following is the most likely mechanism underlying the hypertension?
a. Increased aldosterone
b. Vasodilation
c. Increased blood volume
d. severe vasoconstriction
The paroxysms of elevated blood pressure in a young lady indicates a diagnosis of Phaeochromocytoma.
The diagnosis is confirmed by finding elevated levels of metanephrines and VMA in serum and urine. The
high levels of catecholamines secreted by this tumor causes severe vasoconstriction that causes increased
blood pressure.

39

13. A 20 yrs old man sustains a gunshot injury to his right groin and is treated at the hospital with surgery.
3 months later he presents with a boggy swelling in that region. A thrill is felt over this swelling and a
bruit is detected on auscultation. Which of the following is the most likely cause?
a. A/V fistula
b. Fibrosis
c. surgical scar
d. Aneurysm
The boggy swelling and thrill indicate the formation of an AV fistula secondary to trauma to the vessels.
14. A 35 yrs old man presents to his cardiologist for evaluation of recurrent episodes of lightheadedness and
chest pain for 6 months. one week earlier he got a syncopal(fainting) attack whilst walking up his stairs.
On examination you detect a crescendo-decrescendo midsystolic murmur at the right second intercostal
space and over the carotid artery. Which of the following is this condition often assosiated with?
a. Maternal Rubella infection
b. Tricuspid valve
c. congenital bicuspid valve
d. HCM
This vignette describes a case of Aortic stenosis and AS is more common in patients with a congenital
bicuspid valve. Patients with Turners syndrome also have a higher prevalance of bicuspid valve
15. A 60 yrs old man complaints of fatigue, shortness of breath and pounding in the head for 2 yrs. Past
history is significant for syphilis at age 23.O/E you notice his head bobbing (titubation), prominent neck
pulsations, a waterhammer pulse, quinckes pulse(alternating blanching and erythema of nailbeds) and a
decrescendo diastolic murmur at the left sternal border. Which of the following hemodynamic changes
will maintain his cardiac output?
a. Increased elasticity of aorta
b. Increased left ventricular afterload
c. Increased pulse rate
d. Increased left ventricular preload
AR results in increased preload (EDV) which causes an increased amount of forward blood flow in
subsequent cycles to compensate for the fall in cardiac output.
16. A 33 yrs old female, from India presents with shortness of breath of 6 months duration. Recently she
has been unable to walk up a flight of stairs and last few nights she wakes up suddenly gasping for air.
she was diagnosed with rheumatic fever as a teenager. O/E pulse:100,RR24.JVP is raised, diffuse
crackles heard at lung base, ankle edema bilaterally. Loud S1 and a low pitch rumbling middiastolic
murmur is heard at the mitral area following an opening snap. Which of the following chambers will be
enlarged consequentialy in this patient?
a. Left atrium
b. Left ventricle
c. Right ventricle
d. Right atrium
The diagnosis here is Mitral stenosis and the left atrial chamber gets enlarged consequentialy. Left atrial
pressure is also increased and transmitted to the lungs.

40

17. A 22 yrs old waitress presents to your clinic for evaluation of non exertional chest pain of recent onset.
She appears anxious and gets skipped beats occasionally. O/E you detect a midsystolic click and late
systolic murmur at apex. This condition is assosiated with a disease involving which of the following
tissues?
a. connective tissue
b. Skin
c. Lymphatic tissue
d. Neural tissue
The murmur described is indicative of Mitral valve prolapse and patients with connective tissue disorder
such as Marfans syndrome often have MVP
18. A 50 yrs old diabetic patient presents with an acute MI. On examination, a Pansystolic murmur is
detected at the apex radiating to the axilla. Which of the following complications of an MI has resulted in
this condition?
a. Cardiogenic shock
b. Papillary muscle necrosis
c. Cardiac tamponade
d. septal rupture
e. LBBB
The murmur is typical of MR which is a complication arising from papillary muscle necrosis.
19. An i/v drug addict presents with fatigue, swelling of legs and abdominal bloating. On examination you
detect a raised JVP with large V waves, a holosystolic murmur at the left lower sternal border and a
pulsating liver. Which of the following is the most likely diagnosis?
a. Mitral stenosis
b. Mitral Regurgitation
c. VSD
d. Tricuspid regurgitation
20. A 50 yrs old woman complains of recurrent transient loss of consciousness and dizziness for 3 months.
she has fever and severe wt loss. O/e she is febrile, pale, and has clubbing. A plopping sound is heard in
diastoli followed by a rumbling murmur at the apex. Echocardiogram shows an oscillating lesion near the
mitral valve. Findings vary with body position. What is the most likely diagnosis?
a. Atrial myxoma
b. Bacterial endocarditis
c. Mitral stenosis
d. Austin flint murmur
An atrial myxoma is a benign tumor of the heart and occurs in adults. Rhabdomyomas are the cardiac
tumors that occur in children
21. A 25 yrs old pregnant female patient in her first trimester of pregnancy presents with cough, shortness
of breath and fatigue for 6 months. Cardiac catheterization reveals a PCWP of 30 mm and left
ventricular pressures of 5 mm end diastolic and 130mm systolic. Which of the following is the most
likely diagnosis?
a. Aortic stenosis
b. Aortic regurgitation
c. Mitral regurgitation
d. Mitral stenosis
The raised PCWP implying raised left atrial pressures and decreased left ventricular end diastolic pressure
suggest a diagnosis of Mitral stenosis

41

22. An IV drug addict presents with one week history of fever, rigors and dyspnoea. On examination,
petechial hemorrhage, roth spots, oslers nodes, splinter nail hemorrhage and a new blowing pansystolic
murmur at the apex is auscultated. Echocardiogram shows an oscillating mass attached to the mitral
valve. Which of the following tests is most likely to confirm your diagnosis?
a. Blood culture
b. Echocardiogram
c. ESR
d. CBC
This is a case of Bacterial endocarditis and blood culture is diagnostic.
23. A 65 yrs old man presents with chest pain of acute onset. He is diagnosed with a Myocardial infarct and
4 weeks later is found to have a pansystolic murmur at the lower left parasternal border. Which of the
following is the most likely complication that he has developed?
a. septal rupture
b. Mitral regurgitation
c. ventricular aneurysm
d. Cardiac tamponade
Since the murmur is at the left parasternal border it is a ventricular septal rupture. If it was at the
mitral area it would be Mitral regurgitation due to papillary muscle dysfunction
24. Examination of a patients carotid pulse reveals 2 impulses or peaks in a patient during ventricular
systole. Echo cardiogram reveals a diagnosis of hypertrophic cardiomyopathy. Which of the following
physical findings will also be present?
a. continous machinery murmur
b. mid systolic murmur at left parasternal border
c. Mid diastolic rumbling murmur
d. early diastolic blowing murmur
The pulse described is a bifid pulse which is present in HCM as well as AR. The murmur likely to be present
in patients with HCM is midsystolic ejection murmur at the left parasternal border that becomes louder
with standing and the valsalva maneuvre
25. A 6 months old baby is seen with difficulty in breathing and appears on examination to be floppy and has
weak muscles. Chest x-ray reveals cardiomegaly. which of the following enzyme deficiencies is causal?
a. hyaluronidase
b. acid maltase (alpha glucosidase)
c. amylase
d. elastase
This is the typical presentation of Pompe disease with floppy weak muscles and cardiomegaly. Heart
failure is the final cause of death.
26. During exercise, it is seen that coronary blood flow increases to meet the increased demand of oxygen and
nutrients. This is facilitated by drop in coronary vascular resistance. Which of the following factors will
limit this increased flow during exercise?
a. resting metabolism
b. duration of diastoli
c. contraction force of the myocardium
d. increased demand of myocyte
duration of diastoli decreases as the heart rate increases during exercise. since most of the coronary blood
flow is during diastoli the flow will be limited when the duration of diastoli is reduced

42

27. A young medical student suddenly develops palpitations and light headedness. He had consumed 10 cups of
coffee before this episode. On examination, his pulse is 160/minute and regular, Bp is 110/80 and his
heart sounds are normal. Carotid sinus massage is done and he immediately feels better. HR after
massage is 80/mt. Which of the following mechanisms caused this improvement?
a. Decreased baroreceptor firing
b. prolonging the AV node refractory period
c. increased myocardial refractory period
d. increased sympathetic stimulation of SA node
paroxsysmal supraventricular tachycardia is a common arrythmia that frequently occurs in young people
who have no underlying cardiac problem. The cause is a reentry circuit in the AV node.Episodes are
treated with adenosine in hospital but vagal maneuvres such as carotid sinus massage is also effective.
Carotid sinus massage increases baroreceptor firing thus increasing parasympathetic stimulation and
prolonging the AV node refractory period which stops the reentrant tachycardia
28. A 40yrs old male patient complains of sudden onset palpitations after a party where he consumed a lot
of alcohol. Examination confirms the presence of tachycardia with an irregular rythm. which of the
following EKG findings is most likely in this patient?
a. High voltage QRS complexes in precordial chest leads
b. Prolonged QT interval
c. absent P waves
d. diffuse ST elevations
An irregular tachyarrythmia is most likely due to Atrial fibrillation. In A fib the EKG shows absent p
waves and irregularly spaced ventricular contractions evidenced by variable R-R intervals. P waves are
absent because coordinated atrial contractions do not occur
29. A neonate is seen to be cyanosed and has multiple craniofacial abnormalities, hypoparathyroidism,
hypoplastic thymus and o/e an ejection systolic murmur. You explain to the mother that this condition is
because of abnormal development of neural crest cells. Which of the following is the most commom
congenital heart disease seen in patients with this syndrome?
a. Persistent trunkus arteriosus
b. coarctation of the Aorta
c. PDA
d. Transposition of Great arteries
This vignette illustrates Di George syndrome and the congenital defect indicated is Persistent trunkus
arteriosus which is because of failure of Aorto/pulmonic septal development.
30. A 30 yrs old woman with Marfans syndrome presents with episodes of palpitation,non exertional chest
pain and severe anxiety. Examination reveals a midsystolic click and late systolic murmur at the apex.
Which of the following is the most likely diagnosis?
a. Mitral stenosis
b. Aortic regurgitation
c. Mitral valve prolapse
d. Aortic stenosis
The murmur described is typical of MVP. This condition is freqeuntly present in patients with Marfans
syndrome

43

ICM 2 : Breast Diseases Qstream:


1.

A 23 year old medical student recently read about the high incidence of cancer in women in USA. She is
concerned about her getting breast cancer. What is a risk factor for breast cancer in her?
a. Her mother and aunt had breast cancer in their 50s
b. Diet low in fat content
c. Menarche at the age of 13 years
d. First pregnancy at the age of 19
e. Breast feeding her baby
family history is very important in predicting the breast cancer risk factor

2.

Which of the following patients have the worst prognosis?


a. 5.5 cm bilateral fibroadenoma in a 35 year old
b. 5.5 cm cystosarcoma phyllodes in a nulliparous 45 year old
c. Infiltrating ductal carcinoma in a 47 year old
d. Eczematous lesion right nipple
e. Lobular carcinoma in situ in a 65 year old
infiltrating ductal ca is worst of all cancers in above choices

3.

A 40 year old nulliparous female was found to have a painless hard lump in her left breast. Excisional
biopsy showed infiltrating ductal carcinoma. Sentinel node biopsy showed metastatic involvement with
cancer. Estrogen receptors were negative in the tumor. There was no evidence of cancer elsewhere in the
body. The preferred management is
a. Lumpectomy with axillary node dissection with radiation and chemotherapy
b. Hormonal therapy with simple mastectomy
c. Simple mastectomy
d. Radical mastectomy
e. Radiation to the chest following simple mastectomy

4.

A 47 year old schoolteacher noticed bloody discharge from her left nipple. She denies any history of
trauma. She denies any pain in the breast. The most likely finding in this patient would be
a. Orderly rows of cells with atypia
b. Large nuclei with pale staining cytoplasm
c. Stromal fibrosis with hyperplasia
d. Cellular stroma with plasma cells and lymphocytic infiltration
e. Papillary lesion in lactiferous duct

5.

A 46 year old avid coffee drinker complained of painful lumpy breasts more pronounced at the time of
her periods. Her mammogram showed no suspicious lesion for cancer. Diagnosis of fibrocystic breast
disease was made. She has an increased chance of cancer if there is
a. Increased lymphocytic infiltration
b. More fibrosis than cysts on mammogram
c. Bilateral fibrocystic disease
d. Pop corn calcification on mammogram
e. Atypical ductal hyperplasia

6.

A 45 year old female was found to have a hard 3 cm lump in her right breast. Her right nipple is
inverted. There is dimpling over the lump. Her last menstrual period was two weeks ago. An imaging
study was obtained. The most likely finding would be
a. Filling defect on ductogram
b. Large smooth multiloculated mass on ultrasound
c. Spiculated lesion with pleomorphic calcification
d. Smooth round density on ultrasound
e. Dense breast suggestive of severe fibrosis

44

7.

A 29 year old female who recently gave birth to her first child whom she had been breast feeding till a
week ago when she noticed painful swelling in the left breast. On examination she has redness,
hyperthermia, tenderness and swelling in the left breast. The following statements are true except
a. Biopsy should be done to rule out cancer
b. Infection enters through the cracked nipple
c. E. Coli is the most common pathogen
d. Incision and drainage is the treatment of choice
e. Inflammatory breast cancer should be considered in differential diagnosis

8.

A 75 year old female was recently diagnosed with breast lump. What would suggest the worst prognosis
as far as the breast lesion is concerned?
a. 8cm fibroadenoma
b. Infiltrating ductal carcinoma
c. Bilateral fibrocystic disease
d. Medullary carcinoma
e. Swelling and pain in L4 region

9.

A 65 year old former airhostess was recently diagnosed with breast cancer. The preferred method to
find out the involvement of axillary lymph nodes with cancer is
a. Ultrasound of axilla
b. Axillary node dissection
c. Mammogram
d. Palpation of axilla for enlarged lymph nodes
e. Sentinel lymph node biopsy

10. A 65 year old female was found to have an eczematous lesion on the right nipple. Biopsy showed large
nuclei with pale staining cytoplasm. What is true of this lesion?
a. Bloody nipple discharge is the presenting symptom
b. Usually it is bilateral
c. There is underlying infiltrating ductal carcinoma
d. Ductogram shows a filling defect a.
e. Lump is present in the upper outer quadrant

45

ICM 2 : Gastrointestinal System Qstream:


1.

An 18 yrs old boy presents with abdominal distension,vomiting, and constipation for the past 24 hrs.
On examination of he has an irreducible right inguinal mass. What is the likely cause of this irreducible
mass in inguinal region ?
a. Incarcerated Inguinal hernia
b. Adhesions

2.

A 60 yrs old gentleman presents with severe abdominal pain with bleeding per rectum for the past 6
hours. He had a recent Myocardial Infarction 6 months ago which resulted in coronary artery
angioplasty. His peripheral pulses are diminished in his bilateral lower limbs. What is the possible
diagnosis ?
a. Colonis diverticulum
b. Hemorrhoids
c. Diverticulitis
d. Irritable Bowel Syndrome
e. Ischemic bowel disease
f. Ischemic colitis

3.

A 22 yr old lady presents with abdominal distension, vomiting and constipation for past 24 hours. On
examination abdomen is distended with increased bowel sounds A small inguinal swelling medial to the
Pubic tubercle is detected. X-ray reveals small intestinal obstruction. What is the possible cause of this
obstruction?
a. Incarcerated femoral Hernia
b. Intussusception
c. Adhesions
d. Volvulus
e. Incarcerated Inguinal Hernia

4.

A 40 yr old alcoholic gentleman presents with severe epigastric pain radiating to his back for past 5
hrs. The same pain is relieved by bending forwards. On examination the abdomen is distended with
diminished bowel sounds. What could be the cause for his abdominal pain?
a. Acute Appendicitis
b. Acute cholecystitis
c. Acute pancreatitis
d. Regional ileitis
e. Acute gastritis

5.

A 14 year old boy with cystic fibrosis presents with recurrent attacks of acute pancreatitis. Which of
the below enzymes is first activated in this sequence?
a. Chymotrypsin
b. Lipase
c. Amylase
d. Elastase
e. Trypsin
Inside the acinar cell few molecules of trypsin get activated and these activated molecules activate
trypsinogen (autocatalysis) as well as Chymotrypsinogen producing mole molecules of Trypsin, and
Chymotrypsin causing acute pancreatitis.

6.

Raised levels of which of the below enzymes is specific for diagnosis of acute Pancreatitis?
a. Trypsin
b. Elastase
c. Lipase
d. Cholecystokinin
e. Amylase

46

7.

A 40 yr old female with a body mass index of over 35, presents with pain in right hypochondrium and
nausea for the past 6 hours. She gives a history of a similar although not so severe pain with nausea
following fatty meals for the past 3 months. On examination she is pyrexial. Which of the below clinical
signs is conclusive of the diagnosis?
a. Murphys Sign
b. Gray Turners Syndrome
c. Cullens Sign
d. Rebound tenderness
e. Tenderness all over the abdomen

8.

A 21 yrs old lady presents with resting tremors and new onset psychiatric symptoms. On examination
he is found to have Kayser Fleschers ring on his cornea along with elevated liver function tests. What
dietary restriction are advised to him ?
a. Avoid Poultry
b. Avoid Shell fish and Legumes
c. Avoid Beef products
d. Avoid green leafy vegetables
e. Avoid red wine
Beef live 12.4 mg in 3 ounce serving and Oyster 3.6 mg in 3 ounces serving

9.

A 40 yrs old chronic alcoholic man is brought to the emergency room by his daughter. She says she
found him he was found unconscious in a pool of bright red blood. On examination he has bilateral
gynecomastia, spider angiomata, palmar erythema. What is the cause of his hematemesis?
a. Carcinoma of Stomach
b. Ruptured esophageal varices
c. Reflux Esophagitis
d. Bleeding Peptic Ulcer
e. Mallory Weiss Syndrome

10. A 26 yrs man complains of malaise, and loss of appetite for the past week. On exam you notice he is
jaundiced and labs reveal elevated liver function tests. Further enquiry reveals that his 4yrs old daughter
has recently been kept of her day care due to acute diarrhea although this resolved without treatment.
What is the most likely cause of his jaundice?
a. Hepatitis D
b. Hepatitis B
c. Hepatitis E
d. Hepatitis C
e. Hepatitis A
11. A 35 year old man consults a gastroenterologist because of chronic heartburn for several years. The
heartburn tends to be worse at night, and he frequently tastes refluxed gastric contents when he goes to
bed. He found that his symptoms were a little better when he avoided his customary late evening alcoholic
drink; however, this modest improvement has subsequently deteriorated. The gastroenterologist performs
esophageal manometry with pH monitoring, which demonstrates decreased pressure of the lower
esophageal sphincter and the presence of acid in the esophagus. Esophagogastroduodenoscopy
demonstrates a very irregular gastro esophageal junction with long fingers of reddened mucosa
extending up to 7 cm above the lower esophageal sphincter. Which of the following is the most likely
diagnosis?
a. Esophageal adenocarcinoma
b. Corrosive esophagitis
c. Barrett esophagus
d. Achalasia
e. Seleroderma
f. Esophageal Spasm
g. Zenker diverticulum

47

12. An 8 month old boy with a history of chronic constipation since birth is brought to the emergency room
because of nausea and vomiting. Physical examination shows marked abdominal distension. Abdominal
radiography reveals distended bowel loops with a paucity of air in the rectum. A rectal biopsy shows an
absence of ganglion cells. Which of the following is the most likely diagnosis?
a. Acquired megacolon
b. Imperforate anus
c. Hirschsprung disease
d. Anorectal stenosis
e. Rectal atresia
Hirschsprung disease, also referred to as congenital megacolon, results from a congenital defect in the
innervations of the large intestine, usually in the rectum. Severe chronic constipation is typical. Marked
dilation of the colon occurs proximal to the stenotic rectum, with clinical signs of intestinal obstruction
13. A 60 yrs old gentleman presents with severe left lower quadrant abdominal pain and fever comes to the
E.R. he gives a history of long standing constipation but is otherwise usually well. His WBC are increased
at 18,000. What is the possible diagnosis?
a. Diverticulitis
b. Colonis diverticulum
c. Ischemic bowel disease
d. Hemorrhoids
e. Ischemic colitis
f. Irritable Bowel Syndrome
14. A gastroenterologist performs a colonoscopy on a patient with a family history of gastric and colon
cancer and discovers multiple polyps. Biopsy of one lesion reveals a benign hamartoma. On physical
examination the patient you are likely to note what abnormality:
a. Areas of depigmentation on the hands and face
b. Hyperplastic polyps on the anal mucosa
c. Hyperpigentation of the buccal mucosa and lips
d. Mongolion spot
e. Acanthosis nigricans
The classic clue for Peutz-Jeghers (PJ) syndrome is the melanin pigmentation of the buccal mucosa and
lips (the palms and soles may also be darkened). PJ presents with polyps of the entire gastrointestinal
tract (thus the appearance of colonic lesions), but the small intestine is usually most severely affected. The
polyps of PJ are hamartomas and are not premalignant, although there is an increased incidence of
stomach, breast and ovarian cancer associated with this disease.
15. A 15 year old American Jew with recurrent pulmonary infections and otitis media found to completely
lack the cystic fibrosis transmembrane conductance regulator (CFTR) on epithelial surfaces. he was
diagnosed with meconium ileus when he born. The contents of which of the following lumens is most likely
to be hypertonic?
a. Pancreatic ducts
b. small air ways
c. Bile Ductules
d. Sweat ducts
e. Vas Deferens
@Pancreatic ducts Are hyperactive because of that pancreatic enzynes will be activated

48

16. A 40 year old alcoholic hospitalized after binge drinking with sever abdominal pain and vomiting with
signs of shock and pain was radiating to the back, relieved by bending forwards. Patients condition failed
to improve after initial treatments. Laparotomy reveals a swollen and partially necrotic pancreas.
Activation of which of the enzymes is most likely responsible for this patients condition?
a. Lipase
b. Amylase
c. Elastase
d. Chymotrypsin
e. Phospholipase
f. Trypsin
17. A 76 year old man presents with itchy skin and yellowing of the sclera. He denies pain. On exam you note
a 3 cm mass in the mid epigastruim. You suspect carcinoma of the head of the pancreas. Which of the
following tumor markers would aid in confirming this diagnosis?
a. Alpha-fetoprotein (AFP)
b. Human chorionc gonadotropin (hCG)
c. CA-125
d. Carcinoembryonic antigen (CEA)
e. Prostate-specific antigen (PSA)
Tumor markers can be very helpful in narrowing the possible primary sources for metastatic lesions.
Carcinoembryonic antigen (CEA) can be seen in any tumor derived from gut epithelium, notably colon
cancer and pancreatic cancer. Tumor markers should not be used as the primary tool for cancer diagnosis,
but they have considerable utility in the confirmation of the diagnosis, as well as for monitoring
recurrence or response to therapy.
18. A 70 yr old Asian woman comes complaining of 5kg weight loss and chronic burning epigastric pain for
the last 2 months. This pain is only partly relieved by antacids. You perform endoscopy and note an
invading gastric carcinoma. You are concerned about metastases. Commonly gastric carcinomas spread
across the gastric wall to metastasize to which structure?
a. Pancreas
b. Liver
c. Ovary
d. Gall bladder
e. Diaphragm
Left undiagnosed or untreated gastric carcinomas will eventually extend through the gastric wall to the
serosa and commonly affects the ovary. Involvement of regional and distant lymph nodes also is common,
with metastasis to a left supraclavicular sentinel node frequently the first clinical manifestation of occult
gastric cancer.
19. A 21 year old male is brought to your office with impaired balance and involuntary choreiform
movements. The patient is found to have elevated serum transaminases. Viral Serologies have been
negative. Which of the following would be most helpful in establishing this patients diagnosis?
a. EEG
b. Ct of head
c. Romberg test
d. Audiometry
20. A 65 year old man presents to his local clinic with increasing difficulty swallowing solids. He noticed this
2 months ago and his symptoms have gradually worsened so that recently he even has difficulty with
thicker liquids such as milk shakes. Esophagogastroduodenoscopy is performed and a large tumor is
visualized in the mid-portion of the esophagus. Histology reveals a large squamous cell carcinoma. Which
of the following is a risk factor for squamo cellular carcinoma of the esophagus?
a. Cigarette smoking and alcohol use
b. Frequent use of NSAIDs
c. Diabetes
d. Chronic GERD
e. Celiac disease
49

21. A 16 year old male is brought to your office with impaired balance and involuntary choreiform
movements. The patient is found to have elevated serum transaminases. Viral Serologies have been
negative. Which of the following would be most helpful in establishing this patients diagnosis?
a. Holter moniter
b. audiometry
c. 24 hour EKG
d. Slitlamp examination
e. CT of Head
f. Drug screening
@ young patient abnormal movenents & raised transaminases,& negative viral markes ,one need to
suspect wilsons disease & order for slitlamp examination to see KF rings
22. Laparotomy performed on a 52 year old male who is chronic alcoholic with abdominal pain shows chalk
white lessons in the mesentery. Histologic evaluation of the lesions reveals fat cell destruction and calcium
deposition. This patient most likely suffers from:
a. Crohns disease
b. Bacterial peritonitis
c. Intestinal perforation
d. Celiac disease
e. Acute Pancreatitis
f. Mesenteric ischemia
23. A 40-year old woman presents with intense pruritus and fatigue. Her medical history is significant for
hypothyroidism. Physical examination reveals a slightly enlarged liver and xanthomas. Laboratory studies
show a cholesterol of 538 mgldL and an alkaline phosphatase of 571 lUlL. Which of the following serum
values would most likely be elevated?
a. Serum Cereloplasmin
b. Alpha-fetoprotein
c. Prothrombin time
d. Serum transaminase
e. HBs antigen
f. Antimitochondrial antibody
patient with PBC usually presents with fatigue & pruritis ,anti mitochondrial antibodt is diognostic
24. A 34-year-old woman who is clearly jaundiced is seen in clinic. Laboratory tests reveal conjugated
hyperbilirubinemia. Urine bilirubin levels are significantly above normal, while urine urobilinogen levels are
significantly below normal. Which of the following mechanisms is the most likely cause of her jaundice?
a. Hepatocellular damage
b. Acylouric Jaundice
c. Deficiency of glucuronyl transferase
d. Blockage of the common bile duct
e. Hemolytic anemia
25. A 36-year-old alcoholic man is brought to the emergency department because he is jaundiced and exhibits
inappropriate somnolence. Past medical history is significant for depression and seizures, both of which
are well controlled with medications. Physical exam is significant for icterus, asterixis, and tachypnea. Slit
lamp examination reveals greenish-golden crescents on the cornea. AST is 2489 lUlL, ALT is 4873 lUlL,
bilirubin is 29 mg/dL, alkaline phosphatase is 112 lUlL, hematocrit is 29%, prothrombin time (PT) is
19.1 seconds, and serum ceruloplasmin is decreased. Which of the following is the most likely diagnosis?
a. Alcoholic hepatitis
b. Hepatitis B
c. Budd-Chiari syndrome
d. Wilson disease
e. Druginduced hepatitis

50

26. A 19 year old male patient has severe arthritis involving the lower back. Before making a diagnosis of
ankylosing spondylitis, the patient should be questioned about which of the following diseases?
a. Crohn disease
b. Carcinoid syndrome
c. Whipple disease
d. Celiac disease
e. Peptic ulcer
27. A 45-year-old man presents to his physician with diarrhea, flushing, and wheezing. Physical examination
is significant for a grade VI holosystolic murmur located at the left sternal border at the fourth
intercostal space with no radiation. Which of the following substances is most likely to be elevated in this
patient's urine?
a. Homovanillic acid (HVA)
b. Vanillylmandelic acid (VMA)
c. Selegiline
d. S-Hydroxyindoleacetic acid (S-HIAA)
e. Phenylalanine
in carcinoid syndrome metabalites of serotonin will be increasing
28. A 36 year old native american male is shown to have slow and incomplete gallbladder emptying in
response to cholecystokinin stimulation. This patient is likely to develop;
a. Cystine Stones
b. Uric acid stones
c. Black Pigment stones
d. Phospholipid stones
e. Billiary sludge
f. Brown pigment stones
during formation of any gall stones biliary slde is going to form later forms stones
29. A 74 year old woman is found to have diffuse gallbladder calcification as an incidental finding on
abdominal X-Ray. She denies any history of severe abdominal pain or jaundice. Her vital signs are stable
and physical examination is unremarkable. You inform the patient that
a. The patient is at high risk of gallbladder cancer
b. The patient is prone to get ameboic liver abscesis
c. The patient should be worked up for colon cancer
d. It is a benign finding with no risk of serious disease
e. The patient is at high risk of acute cholecystitis
f. The findings is typically associated with fluke infection of the biliary tree
calcified gall bladder is premalignant
30. A 32 year old male who is having history of intra-venous drug abuse history is hospitalized with lowgrade fever and high serum levels of alaine aminotranserase (ALT) and aspartate aminotranseras (AST).
His serum is positive for HBsAg. You should inform the patient that the most likely outcome for his
infection would be;
a. Fulminant hepatitis
b. Stable chronic hepatitis
c. Complete resolution
d. Chronic hepatitis with progression to corrhosis
e. Chronic hepatitis with progression to hepatocellular carcinoma
93% of hepaties B infction will resolve comlpletely because of the same reason we do not treat them

51

31. The rates of hepatocellular carcinoma secondry to hepatis -B& hepatits-D in some developing countries
are many times higher than that in western nationa. A successful hepatocellular prevention strategy in
these countries would focus in incorporating;
a. Abstinence from Alcohol
b. Universal vaccination
c. Water Sanitation
d. Having only protected Sex
e. Routine physician check-ups
f. Adequate anti-biotic use
if you vaccinate against hepatitis -B virus infection naturally we can eradicate hep-d
32. A 45 year old woman consults a physician because of chronic fatigue. A review of systems reveals
longstanding stomach problems characterized by slow digestion and delayed emptying of her stomach. A
complete blood count demonstrates a moderately severe megaloblastic anemia. Auto antibodies to intrinsic
factor are detected in the serum. A biopsy of the stomach is most likely to show which of the following?
a. Colloid Ca of Stomach
b. Gastric atrophy
c. Acute erosive gastritis
d. Linitis plastica
e. Menetrier disease
f. Peptic ulcer
33. A 42 year old alcoholic presents to the emergency department complaining of 12 hours of epigastric pain
radiating to the back. One and a half hours ago he began to experience nausea and vomiting. His
temperature is 38.4 C (101.2 F), pulse is 100/min, blood pressure is 110/70 mm Hg, and respirations
are 18/min. Abdominal examination reveals mid-epigastric tenderness without guarding, rebound
tenderness, or distension. Laboratory tests show a serum amylase of 1050 U/L, AST of 300 U/L, and a
white blood cell count of 18,000/mm3. a plain film of the abdomen shows several small bowel air-fluid
levels. Which of the following is the most likely diagnosis?
a. Acute gastritis
b. Intestinal obstruction
c. Acute cholecystitis
d. Diverculitis
e. Acute pancreatitis
f. Perforated peptic ulcer
34. A 36 yrs old lady presents with progressive dysphagia to both solids and liquids for 6 months. Endoscopic
examination of the esophagus is normal. Barium swallow examination shows classical Bird beak
pattern with abscnec of peristalsis and failure of the lower esophageal sphincter to relax. What is the
possible diagnosis?
a. Stricture of Esophagus
b. Pharyngeal Pouch
c. Achalasia Cardia
d. Carcinoma of Esophagus
e. Scleroderma
Achalasia is characterized by failure of the lower esophageal sphincter to relax in response to swallowing
and the absence of peristalsis in the body of the esophagus. As a result of these defects in both the
outflow tract and the pumping mechanisms of the esophagus, food is retained within the esophagus and
the organ hypertrophies and dilates. Achalasia is associated with a depletion or absence of ganglion cells
in the myenteric plexuses, which regulate contraction of the esophagus

52

35. A 32 yrs old teacher presents with retrosternal chest discomfort and dry cough for the past 6 months.
She also describes episodic sweating and facial pain radiating to her left arm and the left side of her neck.
She takes over the counter antacid which partially relieves the symptoms. What could be possible cause
of chest pains?
a. Reflux Esophagitis (GERD)
b. Unstable Angina
c. Myocardial infarction
d. Pericarditis
e. Costochondritis
Gastroesophageal reflux can cause aspiration of gastric content into the lung causing cough. It occurs at
night as when patient lies down, the reflux gets exaggerated. GERD is due to incompetent lower
esophageal sphincter (which separates the esophagus from the stomach). Her symptoms arise from the
reflux of gastric contents back into the esophagus, leading to inflammation of the esophageal mucosa. Her
symptoms are exacerbated by large meals, which lead to increase gastric acid secretion, lying in bed, in a
position that allows more of the gastric contents to pass back through the incompetent lower esophageal
sphincter
36. A 40 yrs old gentleman with history of alcoholism and smoking is diagnosed by endoscopy with reflux
esophagitis. He is not taking his prescribed medications. Which of the following complications of reflux
esophagitis is this gentleman likely to get in the long run?
a. Esophageal Rupture
b. Esophageal Web
c. Achalasia cardia
d. Barretts Esophagus
e. Mallory-Weiss Syndrome
Patient with 15 year of GERD have a high likelihood of having Barretts esophagus which is a metaplasia
from squamous epithelium to columnar epithelium which can lead to cancer. Close surveillance with EGD is
indicated even if patient do not have dysphagia.
37. A 46 yrs old gentleman comes to the doctor complaining of weakness and fatigue for the past 12
months. On exam you note he is pale and a full blood count reveals moderate microcystic anaemia. Which
gastro intestinal condition is commonly associated with GI bleeds?
a. Acute Gastritis
b. Pyloric Stenosis
c. Gastric Volvulus
d. Cancer of Duodenum
e. Duodenal Ulcer
Bleeding is the most common complication of peptic ulcer disease, occurring in about 20% of patients.
Bleeding is from the gastroduodenal artery. Chronic blood loss due to occult bleeding is often a feature of
peptic ulcers, whereas massive bleeding occurs less often
38. A 6 week old male infant is brought to the doctors office with projectile vomiting for past 6 hrs. The
vomitus is clear of bile and non-bloody. Examination reveals a dehydrated, fussy infant. Abdominal exam
reveals a walnut sized mass in the epigastrium. What is the possible cause of vomiting?
a. Wilms Tumour
b. Congenital Hypertrophic Pyloric Stenosis
c. Esophageal Atresia
d. Duodenal Atresia
e. Tracheo-esophageal Fistula
Pyloric stenosis classically affects very young neonates and causes projective vomiting.

53

39. A 3 yrs old female child is brought to ED with complaints of progressively worsening abdominal pain.
While in ED child seems to have cramping pains in abdomen every 10 minutes making her draw up her
legs. Rectal examination of the child reveals red currant jelly like stools. What is the possible diagnosis?
a. Ulcerative Colitis
b. Crohns disease
c. Pyloric Stenosis
d. Ileal perforation
e. Intussusception
Abdominal cramping and redcurrent jelly stools are classical of intussusceptions
40. A 40yrs gentleman presents with intermittent episodes of bloody diarrhea over the past three years.
On rectal examination you note multiple fistulae in ano. Colonoscopy reveals skip lesions with rectal
sparring. What is the possible diagnosis ?
a. Ulcerative Colitis
b. Irritable bowel syndrome
c. Amoebic Dysentery
d. Crohns disease
e. Celiac Sprue
41. A 38 yrs lady complains of intermittent bloody diarrhea, weight loss and aphthous oral ulceration for
past 3 years. On colonoscopy she is found to have continous ulcers from rectum to cecum. What is
possible diagnosis?
a. Ulcerative Colitis
b. Amebic Dysentry
c. Irritable Bowel Syndrome
d. Crohns disease
e. Celiac Sprue
42. A 27 yrs old young man complains of intermittent diarrhea for the past two months. The diarrhea
comes on after eating wheat products and is pale and foul smelling. He also describes a weight loss of 10
kgs over the past 2-3 months. Biopsy of the Duodenum reveals villous atrophy with lymphocytic
infiltration. What is the possible cause of his diarrhea?
a. Ulcerative colitis
b. Lymphoma of GIT
c. Crohns disease
d. Celiac Disease
e. Amebic Dysentry
Celiac sprue, which is also referred to as gluten-sensitive enteropathy, is characterized by (1) generalized
malabsorption, (2) small intestinal mucosal lesions and (3) prompt clinical and histopathologic response to
the withdrawal of gluten-containing food. Critical factors in the development of celiac sprue include
genetic predisposition and gliadin exposure. The hallmark of celiac disease is a flat mucosa, with blunting
of villi, damaged epithelial cells, intra-epithelial T cells and increased plasma cells in the lamina propria.
Patients with celiac sprue (gluten enteropathy are at increased risk for the development of intestinal
lymphoma in the gut associated lymphoid tissue. With strict adherence to a gluten free diet, this risk may
be minimized.
43. A 26 yrs old clerk presents with alternating bowel habits (intermittent diarrhea and constipation) and
anxiety neurosis. Abdominal examination is normal except for the presence of peristaltic rush. What is
the possible cause of his problem?
a. Irritable bowel syndrome
b. Intussusception
c. Appendicitis
d. Intestinal obstruction
e. Lymphoma of GIT
Alternating dirrhoea and constipation with loud rushing noises form the bowel is common in IBD

54

44. A 19 yrs old gentleman presents with periumbilical pain, with nausea and vomiting, for the past 2 hours.
On examination patient is found to be febrile with tenderness over the right iliac fossa. Complete blood
count shows a left shift of WBCs. What is the possible diagnosis?
a. Intestinal Obstruction
b. Intussusception
c. Acute Appendicitis
d. Crohns disease
e. Pylonephritis
Abdominal pain wich starts periumbilically and then localizes in the RLQ is often seen in appendicitis.
Look for confirmatory signs such as psoas or obturator signs, as well as rebound tenderness and guarding
45. A 75 yrs old gentleman presents with constipation for past 48 hours. On examination his abdomen is
distended with increased bowel sounds. Rectal examination reveals a hard mass in the rectum. Further
questioning reveals alternating diarrhea and constipation for past 6 months. What is the cause of his
intestinal obstruction?
a. Anal fissure
b. Hemarrhoids
c. Rectal Polyp
d. Carcinoma rectum
e. Rectal Stricture
46. A 27 yrs old woman presents with painful defecation with bleeding 6 weeks after delivery of a healthy
male infant. She describes the blood as bright red and found on the toilet paper and side of the bowl.
Rectal examination reveals a linear ulcer on the verge of the anus with induration on the posterior verge
of the anus. What is the most possible diagnosis?
a. Carcinoma in Rectum
b. Fissure in Ano
c. Crohns disease
d. Fistula in Ano
e. Hemorrhoids
47. An 18 yrs old boy presents with abdominal distension, vomiting, and constipation for the past 24 hrs. On
examination of he has an irreducible right inguinal mass. What is the likely cause of this irreducible mass
in inguinal region?
a. Incarcerated Inguinal hernia
b. Adhesions
c. Colonic cancer
d. Intussception
e. Diverticulitis

55

You might also like